SlideShare a Scribd company logo
1 of 30
Download to read offline
Basic Medical Science, March 2007
สหพันธสิตนักศึกษาแพทยแหงประเทศไทย
National Council of Thai Medical Students (NCTMS)
www.nctms.in.th
-1-
แนวขอสอบ
Basic Medical Science
March, 2007
สหพันธนิสิตนักศึกษาแพทยแหงประเทศไทย
ประจําปการศึกษา 2550
รุนที่ 17
1. ผูหญิงคนหนึ่งไปเสริมนม silicone ทําแลวนมผิดรูป
อาการนี้เกิดจากเซลลใดเกี่ยวของ
ก. T lymphocyte
ข. B lymphocyte
ค. Histiocyte
ง. PMN
จ. Basophil
2. ชายอายุ 50 ป ดื่มสุราเปนประจํามาเปนเวลา 10 ป มี
อาการปวดทองบริเวณใตลิ้นป บางครั้งปวดราวทะลุไป
ดานหลัง ถายอุจจาระเปนมูกมัน ไมมีตาเหลือง ตาเหลือง
อาการดังกลาวสัมพันธกับพยาธิสภาพที่อวัยวะใด
ก. liver
ข. stomach
ค. pancrease
ง. duodenum
จ. gall bladder
3. เด็กชายอายุ 10 ป กลืนลําบากเนื่องจาก lower
esophagus sphinter ทํางานผิดปกติ ถาทํา manometry
LES pressure จะเปนอยางไร
ก. สูงกวาปกติในพัก
ข. สูงกวาปกติในขณะกลืน
ค. ต่ํากวาปกติในขณะผัก
ง. ต่ํากวาปกติในขณะกลืน
จ. ต่ํากวาปกติในกลืน และสูงกวาปกติในขณะพัก
4. เด็กชายอายุ 3 ป ผิวแหง ผิวลอก เปนโรคขาดสารอาหาร
แบบ kwarsiokor อยากทราบวาจะมี amino acid ตัวไหน
ปกติ
a. Threonine b. Tryptophan
c. Methionine d. Aspartic acid
e. Phenyl alanine
5. การทํา mediolateral episiotomy หลีกเลี่ยงอันตราย
ตอ muscle ใด
ก. bulbospongiosus
ข. ischicavernosus
ค. superficial peroneal nerve
ง. deep peroneal nerve
จ. external anal sphinter
6. ผูปวยหญิงอายุ 17 ป มาพบแพทยเนื่องจากไมมี
ประจําเดือน มีขนาดหนาอกปกติ pubic hair ปกติ ตรวจ
ชองคลอดพบวามี blind vaginal pouch และมีกอน
เคลื่อนที่ไดบริเวณ labia majora ตรวจ ultrasound ไมพบ
uterus ผูปวยนาจะเปนโรคใด
a. Turner syndrome
b. Gonadal dysgenesis
c. Adenogenital syndrome
d. Female pseudohermaphrodite
e. Testicular feminizing syndrome
Basic Medical Science, March 2007
สหพันธสิตนักศึกษาแพทยแหงประเทศไทย
National Council of Thai Medical Students (NCTMS)
www.nctms.in.th
-2-
7. ผูปวยเด็ก 5 ป เปน glioma ขนาด 4 cm ที่ Rt. Frontal
lobe ไดรับการฉายรังสี กอนเล็กลงเล็กนอย อะไรทําใหดื้อ
ตอการรักษา
ก. inactivation of bcl2 gene
ข. inactivation of p53 gene
ค. inactivation of precursor of MDL-2 gene
ง. activation of BCL-XL gene
จ. activation of retinoblastoma gene
8. คนไขชายมีอาการหอบ ตามัว จากการดื่มสุราเถื่อน
หมอให ethanol แกผูปวยเพื่อรักษา กลไกการออกฤทธิ์
ของ ethanol คือ
ก. Product inhibitor
ข. Allosteric inhibitor
ค. Irreversible inhibitor
ง. Competitive inhibitor
จ. Feedback inhibitor
9. กลไกของยา cyclosporine จะยับยั้งการทํางานของ
cell ใด
ก. B-cell
ข. T-cell
ค. Macrophage
ง. NK cell
จ. PMN
10. เด็กชายอายุ 2 ป เดินชนสิ่งของบอยๆ ตอนกลางคืน
ตรวจรางกาย พบ Biot’s spot (เกล็ดกระดี่) ที่ตา อาการ
ของผูปวยเกิดจากการขาดสารในขอใด
ก. Retinol
ข. Thiamine
ค. Riboflavin
ง. Pyridoxine
จ. Ascorbic acid
11. Urinalysis ตรวจตอนไหนเปนอยางชาจะไดผล lab ที่
เชื่อถือได ถาเก็บตอน 14.00 น.
ก. 14.30 ข. 15.00
ค. 16.00 ง. 18.00
จ. 20.00
12.หญิงอายุ 60 ป อาเจียนอยางรุนแรงและถายอุจจาระ
เปนน้ํา 5 ครั้ง ใน 6 ชั่วโมง ตรวจรางกายพบ postural
hypotension ริมฝปากแหง poor skin turgor รางกายของ
ผูปวยจะมีการเปลี่ยนแปลงอยางไร
a. การหลั่ง aldosterone ลดลง
b. ระดับ plasma angiotensin II ต่ํา
c. การหลั่ง ANP เพิ่ม
d. Sympathetic activity เพิ่มขึ้น
e. Efferent arteriole resistant ที่ไตเพิ่มขึ้น
13. เด็กหญิงอายุ 2 ป ถายอุจจาระเปนน้ําวันละ 10 ครั้ง
มา 2 วัน ตรวจ serum CO2 = 12 mmol/L urine pH= 5.0
การเปลี่ยนแปลงทางหองปฏิบัติการดังกลาว เกิดจากการ
compensation ที่ตําแหนงใดของ nephron
a. Glomerulus
b. Proximal tubule
c. Loop of henle
d. Distal tubule
e. Collecting duct
14. พิษสุราเรื้อรัง ชาปลายมือปลายเทา deep tendon
reflex แขนขาลดลง metabolic pathway ใดผิดปกติ
a. glycolysis
b. Kreb’s cycle
c. Glucogenesis
d. Glycogenesis
e. Oxidative
Basic Medical Science, March 2007
สหพันธสิตนักศึกษาแพทยแหงประเทศไทย
National Council of Thai Medical Students (NCTMS)
www.nctms.in.th
-3-
15. ผูปวยเด็กเปน acute lymphoblastic leukemia ไดรับ
ยาเคมีบําบัด มีภาวะ uric acid สูง metabolism สารใด
ผิดปกติ
a. heme
b. purine
c. pyrimidine
d. amino acid
e. glucoronide
16. ผูปวยชายอายุ 55 ป ถายปสสาวะเปนสีเลือด จากการ
ตรวจชิ้นเนื้อ พบวาเปน transitional cell carcinoma ที่
กระเพาะปสสาวะ ผูปวยคนนี้นาจะมีพฤติกรรมตรงกับขอ
ใด
a. สูบบุหรี่จัด
b. อั้นปสสาวะบอย
c. ประกอบอาชีพเกษตรกรรม
d. ดื่มสุรามาก
e. ทานอาหารปงยาง
17. ชายอายุ 50 ป ดื่มเหลาเปนประจํา 10 ป ปวดบริเวณ
epigastric ราวไปที่หลัง ถายเปนมูกมัน ตับมามไมโต มี
ความผิดปกติที่อวัยวะไหน
a. stomach b. liver
c. pancrease d. small intestine
e. gallbladder
18. ชายอายุ 15 ป เตะฟุตบอล หายใจเร็วและแรงขึ้น
กลไกใดทําให ไดรับอากาศใหมมากที่สุด
a. Tidal volume
b. Respiratory rate
c. Minute ventilation
d. Dead space ventilation
e. Inspire reserve volume
19. ชายอายุ 50 ป วินิจฉัยเปน pleural effusion ตองเจาะ
effusion มาตรวจ โดยเจาะระหวาง ant. Axillary line กับ
pleural reflexion ตําแหนงที่เจาะอยูระดับ rib ใด
a. 7
b. 8
c. 9
d. 10
e. 11
20. ชายอายุ 35 ป หลังอดน้ําและอาหารเปนเวลา 16 ชม.
เพื่อเตรียมรับการผาตัด มีระดับ urine sp.gr 1.025
อยากทราบวาระดับ hormone ใดในเลือดเพิ่มขึ้น
a. insulin
b. Oxytocin
c. Vasopressin
d. Glucocorticoid
e. ANP
21. หญิงไทยมีอาการออนเพลีย ปวดขอ เปนแผลที่ปาก
เปนๆหายๆ มาเปนเวลานาน ตรวจพบ antinuclear ab
titer 1:512 ควรตรวจอะไรเพิ่มเพื่อ diagnosis โรค
a. anti ds DNA ab.
b. SER
c. CD4 T-cell
d. CBC
e. ANA
22. อัตราการตายของผูใหญอายุ 70 ป เปน 0.1/ป อัตรา
การตายของผูปวยโรคหัวใจเปน 0.08/ป อัตราการตายของ
ผูปวยโรคมะเร็งลําไสเปน 0.02/ป ถาม life expectation
ของผูใหญอายุ 70 ป ที่เปนโรคหัวใจและมะเร็งลําไส
a. 1 b. 2
c. 5 d. 8
e. 10
Basic Medical Science, March 2007
สหพันธสิตนักศึกษาแพทยแหงประเทศไทย
National Council of Thai Medical Students (NCTMS)
www.nctms.in.th
-4-
23. ความสัมพันธระหวาง FEV1 (force expiration in 1
second) กับอายุ เปนดังสมการ FEV1(liter) = (1.052 x
height) - (0.244 x age) -0.561 ขอใดกลาวถูกตอง
a. ถาอายุขัยลดลง 1 ป ,FEV1 จะลดลง 0.244 liter
b. ถาอายุขัยลดลง 0.244 ป FEV1 จะลดลง 1 liter
c. ถาอายุเพิ่มขึ้น 1ป FEV1 จะลดลง 0.244 liter
d. ถาอายุเพิ่มขึ้น 0.244 ป FEV1 จะลดลง 1 liter
24. เด็กชายอายุ 5 ป มีความผิดปกติของ LES (lower
esophageal sphincter) ถาทํา esophageal manometry
จะพบภาวะใด
ก. ต่ําลงขณะพัก
ข. ต่ําลงขณะกลืน
ค. เพิ่มขณะพัก
ง. เพิ่มขณะกลืน
จ. เพิ่มขณะพัก&ลดขณะกลืน
25. ชาย 25 ป ฝกทหารกลางแดดเปนเวลา 2 ชั่วโมง
ติดตอกัน มีอาการหนาแดง ตัวรอน ตัวแหง ไมมีเหงื่อและ
หมดสติ ควรทําอยางไรเปนการรักษาอาการไดดีที่สุด
a. ใชพัดลมเปา
b. ใหนอนยกเทา
c. ใชน้ําเย็นราดใหโชกทั้งตัว
d. กันไมใหคนมุง
e. ใชผาหอน้ําแข็งวางบนหนาผาก
26. ชายอายุ 50 ป เสียชีวิตในรถยนตที่ติดเครื่องและเปด
เครื่องปรับอากาศ สภาพศพผิวหนัง มีสีแดงสด ญาติเลา
ประวัติวา ผูตายนอนในรถนานกวา 5 ชม. สภาพผิวหนังสี
แดงสด เกิดจาก Hb อะไร
a. sulfhemoglobin b. methamoglobin
c. oxyhemoglobin
d. carboxyhemoglobin
e. oxidized hemoglobin
27. สูบบุหรี่มานาน หอบเหนื่อยงายขอใดถูก
a. FEV1/FRC ลด
b. Elascity ลด
c. Compliance ลด
d. Restrictive fibers
28. ชายอายุ 20 ป ปวดที่ขอสะโพก แพทยพบวาเปน
posterior dislocation จงบอกวามีการเคลื่อนไหวอยางใด
ที่ hip joint จึงทําใหเกิดอาการขึ้น
a. flex, medial rotate, adduct
b. flex, medial rotate, adduct
c. flex, lateral rotate, abduct
d. extend, lateral rotate, abduct
e. extend, medial rotate, abduct
29. ชายอายุ 25 ป มีโพรงหนองที่แขนและนอง แพทย
วินิจฉัยวาเปน polymyositis เจาะหนองออกมายอมเชื้อ
พบวาเปน bacterial ทรงกลม ทดสอบ coagulase และ
catalase ใหผลเปนบวก นาจะเปนเชื้อใด
a. Stap. Aureus
b. Stap. Epidermidis
c. Stap. Haemolyticus
d. Strep pyogenes
e. Strep. Pneumoniae
30. กินกลวยบวดชีกอนบริจาคเลือด พบวาเลือดขุนขน
จากอะไร
a. micelle
b. chylomicron
c. cholesterol
d. fatty acid
e. monoglycerine
Basic Medical Science, March 2007
สหพันธสิตนักศึกษาแพทยแหงประเทศไทย
National Council of Thai Medical Students (NCTMS)
www.nctms.in.th
-5-
31. แพทยทําคลอด สังเกตวามีน้ําคร่ํามากกวาปกติ
หลังจากนั้น 1 วัน มีอาการสําลักนม และไมสามารถกลืน
อาหารได ไมสามารถใสทออาหาร( NG tube ) มีความ
พิการแตกําเนิดอะไร
a. midgut volvulus
b. duodenal atresia
c. annular pancrease
d. hypertrophic pyrolic stenosis
e. esophageal atresia
32. ผูชายอายุ 30 ป tidal volume 500 mL,anatomical
dead space 200 mL, RR15 ถาเปลี่ยน RR เปน30,tidal
volume 350 แต anatomical dead space เทาเดิมจะมี
ผลเปลี่ยนแปลงอยางไร
a. Paco2
b. Paco2
c. paco2ไมเปลี่ยนแปลง
d. pH
e. pHลด
33. ชายอายุ 70 ป สูบบุหรี่มา 40 ป มีอาการไอเรื้อรัง
บางครั้งหอบมา2ป x-ray ปรากฏวาเปน emphysema คา
ใดจะลดลง
a. airway resistance
b. lung compliance
c. intrapleural P
d. FRV
e. FEV1
34. ถารางกายขับ bile salt มากขึ้นถามวา สารในขอใด
ในรางกายลดลง
a. bilirubin
b. lecithin
c. fatty acid
d. triglyceride
e. cholesterol
35. ผูปวยชายอายุ 50ป สูบบุหรี่จัด มาที่หองฉุกเฉินดวย
อาการเจ็บหนาอกมา 1 ชั่วโมง สง lab พบวา creatinine
kinase และ troponin-t เพิ่มขึ้น ถามวาเกิดอะไรขึ้น
a. dissociation of ribosome
b. swelling ของmitochodria
c. clumpingของ nuclear chromatin
d. Fragmentation ของ plasma membrane
36. ชายอายุ 30ป ไมมีโรคประจําตัว ปวดศีรษะอยางรุนแรง
ขณะทํางาน ไมเคยปวดแบบนี้มากอน ไมมีอาการนํามากอน
ตรวจรางกายพบ stiff neck และ papilledema ไมมีอาการ
muscle weakness ลักษณะแสดงถึงพยาธิสภาพใด
a. brain abscess
b. hydrocephalus
c. Thrombotic stroke
d. acute bacterial meningitis
e. Ruptured cerebral aneurism
37. หญิงอายุ60ป มีประวัติเปนความดันโลหิตสูงและ
เบาหวานมา10ป หลังตื่นนอนมีอาการแขนขาขวาออนแรง
และพูดไมได ตรวจรางกายพบright hemiparesis with
motor aphasia มีการอุดตันที่หลอดเลือดใด
a. anterior cerebral artery right
b. anterior cerebral artery left
c. middle cerebral artery right
d. middle cerebral artery left
e. posterior cerebral artery right
38. ผูปวยหญิงตรวจรางกาย ที่ปากมดลูกพบ red
vesicle,erythema and papule เปนกลุมๆคลําพบตอม
น้ําเหลืองโต 2 ขาง กดเจ็บ ตรวจ ทาง histopathology พบ
intranuclear halo ,multinucleated giant cell เชื้อที่
เปนไปไดคือ
a. HSV b. Gonococcal infection
c. CMV d. Treponema pallidum
e. HPV
Basic Medical Science, March 2007
สหพันธสิตนักศึกษาแพทยแหงประเทศไทย
National Council of Thai Medical Students (NCTMS)
www.nctms.in.th
-6-
39. นายวิชัย และนายวิวัฒนวิ่งแขงขันกีฬามหาวิทยาลัย
โดยนายวิชัยแขงวิ่ง 200 m นายวิวัฒนแขงวิ่งมาราธอน
ถามวาระดับปริมาณของสารใดในกลามเนื้อขาของนาย
วิชัยที่มีมากกวานายวิวัฒน
ก. จํานวน mitochondria
ข. ระดับglycolytic enzyme
ค. muscle blood flow
ง. Fat composition
40. หญิง 3 5ป มีกอนที่หนาอกขวามา1m กอนโตขึ้น กด
เจ็บ กอนมีขนาด 6 cm hard consistency มี bloody
discharge ออกจาก nipple คลําที่ lymph node ดานขวา
พบกอนขนาด 4cm ขอใดบงลักษณะของมะเร็ง
ก. กอนขนาด 6 cm
ข. กดเจ็บ
ค. มี bloody discharge ออกมา
ง. คลําพบกอนที่ LN
จ. Hard consistency
40. ขอใดมีโอกาสเกิด resolution มากที่สุด
ก. ruptured spleen
ข. skin ulcer
ค. lobar pneumonia
ง. bowel gangrene
จ. chronic viral hepatitis
41. ผูปวยหญิงอายุ 35 ป ตาโปน ใจสั่น เหนื่อยงาย
diffuse enlargement of thyroid gland, free T4 สูงขึ้น
การตรวจ lab ใดสําคัญที่สุดในการวินิจฉัยโรค
ก. thyroid scan
ข. triiodothyroxine
ค. thyroxine binding gobulin
ง. TSH
จ. TRH
42. ชายอายุ 35 ป ไมเคยออกกําลังกาย ออกกําลังกาย
ครั้งแรกเสียเหงื่อมาก กอนออกกําลังและหลังออกกําลังมี
อาการปกติ ถามวา rennin-angiotensin system มีกลไก
การรักษา BP อยางไร
a. เพิ่ม GFR
b. เพิ่ม renal blood flow
c. เพิ่ม Na reabsorption
d. ลด ACE activity
43. หญิง BP ตก มีประวัติทานยาหมอ ตรวจรางกายอื่นๆ
มี buffalo hump หากเจาะดูจะพบ serum electrolyte
ยังไง
ก. hyponatremia
ข. hypokalemia
ค. hypocalcemia
ง. metabolic alkalosis
จ. hyperuricemia
44. ผูชายมาดวยอาการปวดทองนอยขวา ตรวจพบ
acute appendicitis ซึ่งมีการ inflammation แบบใด
a. serous inflammation
b. organized inflammation
c. fibropurulent inflammation
d. granulomatous inflammation
45. มี abscess ที่ขาขวานํามาตรวจพบ Gram +ve cocci
in cluster และมี Lymph node ที่ขาหนีบโต มี pathology
แบบใด
a. sinus histiocytosis
b. follicular hyperplasia
c. parafollicular hyperplasia
d. granulomatous hyperplasia
e. interfollicular hyperplasia
Basic Medical Science, March 2007
สหพันธสิตนักศึกษาแพทยแหงประเทศไทย
National Council of Thai Medical Students (NCTMS)
www.nctms.in.th
-7-
46. หญิง 30 ป เสริมหนาอกดวย silicone 1 ปตอมา คลํา
ไดกอนแข็งที่เตานมขางขวาและเตานมผิดรูป เซลลใดที่มี
บทบาทสําคัญที่ทําใหเกิดพยาธิสภาพนี้
a. Basophil
b. Eosinophil
c. Histiocyte
d. Neutrophil
e. Plasma cell
47. ชายอายุ 17 ป รางกายปกติ กินเนื้อเค็มและน้ําตาม
a. ลดการกระตุน Baroreceptor
b. ลดการกระตุน Low Pressure receptor
c. เพิ่ม rennin
d. ลดการกระตุน chemoreceptor
e. เพิ่ม Hydrostatic pressure
48. เมื่อเรากินเค็ม อะไรเพิ่ม
a. Na+
เพิ่ม
b. K+
เพิ่ม
c. Renin เพิ่ม
d. Aldosterone เพิ่ม
e. ANP เพิ่ม
49. ยา Nanosum ไดรับอนุญาตใหวางขาย ตอมาพบวา
คนใชยาไดรับผลขางเคียง ขั้นตอนใดที่สามารถระงับการ
วางจําหนายตอไปได
a. Phase IIa Clinical trial
b. Phase IIb Clinical trial
c. Phase III Clinical trial
d. Phase IV Clinical trial
50. เด็ก 2 ป ถายเปนน้ํา serum CO2 Urea pH 5.0
nephron มี compensate ที่ไหน
ก. Glomerulus
ข. Proximal tubule
ค. Loop of Henle
ง. Distal tubule
จ. Collecting duct
51. หญิงหมาย 50 ป หมดประจําเดือนมี Fasting blood
glucose 200 Triglyceride 200 HDL 35 LDL 150 จะ
เสี่ยงตอการเปนโรคอะไร
a. Alzheimer
b. Depression
c. Coronary heart disease
d. Cerebrovascular disease
52. ชายอายุ 40 ป ไมมีโรคประจําตัว พบ Hct 60% WBC
6,000 platelet 200,000 สัมพันธกับขอใด
ก. อวน
ข. ดื่มสุรา
ค. สูบบุหรี่
ง. ยาฆาแมลง
จ. ทํางานในโรงงานอุตสาหกรรม
53. ชายอายุ 20 ป ถูกแทงที่หนาทอง แพทยไดทําการ
ผาตัดหนาทองลงไปถึงชั้น transverse abdominis อยาก
ทราบวาโครงสรางใดที่อยูถัดจากชั้นนี้
a. external abdominis muscle
b. Internal abdominis muscle
c. Rectus abdominis
d. Transversalis fascia
e. Peritoneum
Basic Medical Science, March 2007
สหพันธสิตนักศึกษาแพทยแหงประเทศไทย
National Council of Thai Medical Students (NCTMS)
www.nctms.in.th
-8-
54. ชายอายุ 20 ป ดําน้ําดูปะการัง กอนดําหายใจลึก 5-6
ครั้ง คอยๆขึ้นผิวน้ําชาแลวหมดสติ ถามวาเพราะอะไร
ก. Hypoxia ข. Hyperoxia
ค. Hypocapnia ง. Hypercapnia
จ. Pneumothorax
55. ชาย 25 ป รถชน หมดสติ หัวมีบาดแผล ตรวจ
รางกายได HR 66 RR 20 Pressure 210/100 ถามวา
ปจจัยทาง CVS ใดที่มีผลตอความดันที่สุด
ก. Blood volumn
ข. Preload ของ หัวใจ
ค. Afterload ของ หัวใจ
ง. Sym ที่หัวใจ
จ. Peripheral Vasoconstriction
56. ชาย 20 ป ไขต่ํา 8 สัปดาห ตรวจรางกายพบ
murmur ตรวจ echo หัวใจ พบ 0.5 cm vegetative ที่
mitral เพาะเชื้อ 7 วัน ไมขึ้น Bacteria ถามวาเชื้อใด
ก. E.faecalis ข. S.Aureus
ค. H.aphrophilus ง. P.aeruginosa
จ. S.epidermidis
57. บริเวณที่ใชเปนตําแหนงทํา Bone Marrow biopsy ใน
ผูใหญ อยูที่ใด
a. Tibial b. Humerus
c. Radius d. Femur
e. Iliac spine
58. ยาเกามีประสิทธิภาพในการรักษา 60 ยาใหมมี
ประสิทธิภาพ 80 คา Number needed to treat เทาไหร
ก. 5 ข. 20
ค. 33.3 ง. 50
จ. 10
59. ผูชายอายุ 25 ป มาดวยอาการไข ไอหายใจเหนื่อย
หอบ มา 2 วัน แพทยตรวจเสมหะพบ Gram +ve
diplococci แพทยใหการวินิจฉัยวาเปน Pneumonia
Criteria ที่ประเมินวา sputum gram strain นี้นาเชื่อถือ
Epithelium (/HPF) WBC(/HPF)
a. 10 40
b. 25 30
c. 30 50
d. 50 10
e. 30 40
60. ชาย 50 ป เปนโรคตับแข็ง มี Ascites ไดรับยา
furosemide 1 wk ตอมามีอาการออนแรง เวียนศีรษะ
สับสน เปนตะคริว ตรวจเลือดผล Na 130 K 2.5 pH 7.58
PCO2 50 ความผิดปกติสมดุล acid-base เปนแบบใด
a. uncompensated respiratory alkalosis
b. compensated respiratory alkalosis
c. uncompensated metabolic acidosis
d. compensated metabolic acidosis
e. mixed respiratory and metabolic alkalosis
61. ชาย 25 ป ไข ไอหอบ มา 2 เดือน หมอวินิจฉัยวาเปน
pneumonia ยอม gram พบ positive diplococci ดูความ
นาเชื่อถือ ของเชื้อใน sputum วาอยางไร adequate
62. Terbutaline sulfate ออกฤทธิ์ยังไง
a. alpha1 antagonist
b. alpha2 agonist
c. beta1 agonist
d. beta2 agonist
e. antimuscarinic
Basic Medical Science, March 2007
สหพันธสิตนักศึกษาแพทยแหงประเทศไทย
National Council of Thai Medical Students (NCTMS)
www.nctms.in.th
-9-
63. มีอาการ แขนขาออนแรง right hemiparesis + motor
aphasia เสียที่เสนเลือดใด
a. ant. Cerebral a. right
b. right middle cerebral a.
c. right post. Cerebral a.
d. left ant. Cerebral a.
e. left middle cerebral a.
64. ทารกแรกเกิดมีอาการซีด บวมน้ําอยางมาก ตับมามโต
ตรวจพบวารกมีขนาดใหญ ผิดปกติขณะคลอด มารดามี
อาการครรภเปนพิษ ตรวจ VDRL ใหผลลบ ทารกถึงแกกรรม
หลังคลอด 2 ชม. ผลการตรวจ Hb typing ไดผลเปนเชนใด
a. alpha 4 b. beta 4
c. gamma 4 d. delta 4
e. zeta 4
65. ชายอายุ 50 ป เปนไตวายเรื้อรัง ตรวจรางกายพบซีด
บวมขาทั้ง 2 ขาง คลํา ตับมามและตอมน้ําเหลืองไมพบ
ตรวจเลือดพบ Hb 8 g/dl Hct 24% WBC 6000 N75%
L25% platelet 200000 MCV 80 fL reticulocyte 1%
creatinine 5 อาการซีดของผูปวยรายนี้ เกิดจากการขาด
สารใดมากที่สุด
a. Folic acid b. Erythropoietin
c. Thyroid hormone d. Cyanocobalamine
e. Colony stimulating factor
66. หญิง 15 ป มา ER ดวยอาการ หายใจและกลืนลําบาก
แขนขาไมมีแรงเห็นภาพซอนหลังจากกินผัดกาดดองกระปอง
มา 10 hr ลักษณะทางคลินิกดังกลาวมาจากอะไร
a. ยับยั้งการหลั่ง Ach
b. ยับยั้งการหลั่ง NE
c. ยับยั้ง AchE
d. กระตุน muscarinic Receptor
e. กระตุน adrenergic receptor
67. ชายอายุ 50 ป เดินทางจาก กทม. ไป New York city
ตอมามีอาการออนเพลีย นอนไมหลับ ระบบทางเดิน
อาหารแปรปรวน อาการดังกลาวสัมพันธ กับ hormone
จากโครงสรางตอไปนี้
a. anterior Pituitary
b. posterior pituitary
c. pineal gland
d. hypothalamus
e. adrenal gland
68.มารดาตั้งครรภ 28 สัปดาห คลอดทารกกอนกําหนด
ทารกตัวเขียว ถูกวินิจฉัยวาเปน respiratory distress
syndrome ให surfactance ไปแลว อาการดีขึ้น
surfactant ที่ใหไป ออกฤทธิ์อยางไร
a. เพิ่ม lung compliance
b. ทําใหขนาดของถุงลมเทากัน
69. ผูปวยอายุ 70 ป มีอาการความจําเสื่อม แพทยวินิจฉัย
วาเปน Alziemer สมองเสื่อมที่ใด
a. cerebropontine
b. cerebellum
c. hippocampus
d. medulla oblongata
e. globus pulidus
70. ผูปวยเด็ก 5 ขวบ มีอาการไขต่ําๆ มา 1 เดือน พอเปน
TB ทํา tuberculin test ในผูปวยเด็ก positive ถามวาผล
ของการทํา tuberculin test เกิดจากปฎิกิริยาใด
a. hypersensitivity type 1
b. hypersensitivity type 2
c. hypersensitivity type 3
d. delay type hypersensitivity
e. none of above
Basic Medical Science, March 2007
สหพันธสิตนักศึกษาแพทยแหงประเทศไทย
National Council of Thai Medical Students (NCTMS)
www.nctms.in.th
-10-
71. ชายอายุ 30 ปมาดวยอาการหนังตาตก มีกลามเนื้อ
ออนแรง ผูปวยมีอาการหนักมากขึ้นตอนเย็น มีตอม
thymus โต กลไกการเกิดโรคเปนอยางไร
a. immediate hypersensitivity
b. delay type hypersensitivity
c. immune complex hypersensitivity
d. Ab-cytotoxic hypersensitivity
e. Complement activation
72. เด็กทารกอายุ 2 วัน ชักเกร็งหลังแอนเปนพักๆ ตั้งแต
แรกคลอด มารดาเปนชาวไทยภูเขา ไมไดฝากครรภ ครบ
กําหนดคลอด หมอตําแยทําคลอดโดยใชไมรวกติดสาย
สะดือ เด็กคนนี้เปนอะไร
a. septicemia b. birth trauma
c. hydrocephalus d. status epilepticus
e. tetanus neonatorum
73. ชาย 50 ป มีตุมน้ําพุพอง ตามลําตัวและแขนขามา 1
เดือน ตรวจผิวหนังพบ vesicobullous eruption ทั่วทั้งตัว
ทํา skin biopsy พบ acantholytic cell ที่ suprabasal
bullae โครงสรางใดที่ผิดปกติ ( มีรูป )
a.microtubule b.desmosome
c.keratin filament d.basement membrane
e.keratohyaline granule
74. ชายอายุ 65 ป ไดรับการวินิจฉัยวาเปน alzheimer
ถามวาอะไรเปนปจจัยที่สําคัญที่สุดที่ทําใหเซลลประสาท
ถูกทําลาย
ก. nerve growth hormone ลดลง
ข. intracellular ca2+ ของเซลลประสาทลดลง
ค. glycogenolysis ในเซลลลดลง
ง. gluconeogenesis ในเซลลลดลง
จ. ......
75. หญิงอายุ 25 ป ตั้งครรภครบกําหนดคลอดแลวเกิด
อาการเจ็บทองคลอด ผลที่เกิดขึ้นนี้เกิดจาก Hormone ใด
1) Prolactin 2) Oxytocin
3) Vasopressin 4) Catecholamine
5) Corticosteroid
76. คนไขหญิงอายุ 35 ป มีอุณหภูมิรางกายสูง มูกปาก
มดลูกเปนลักษณะเหนียวขน ลักษณะฮอรโมนในขอใดที่
ถูกตอง
1. FSH สูง 2. testosterone สูง
3. estrogen ต่ํา 4. Progesterone สูง
5. Estrogen สูง
77. ชายอายุ 25ป ปสสาวะแสบขัด มีหนองไหลออกจาก
ทอปสสาวะ นําdischargeไปยอมแกรมพบintracellular
gram negative diplocci มีประวัติแพpenicillin ควรใหยา
อะไรในการรักษา
a. Imipenam b. Gentamicin
c. Erythromycin d. Ciplofloxacin
e. Bactrim
78.เด็ก 10 ขวบ ลมกระดูก humorus หัก สงผลใหเหยียด
ขอมือกับนิ้วมือไมได มีสภาวะแทรกซอนที่ใด
1) median nerve 2) radial
3) Ulnar
4) medial cord of branchial plexus
5) lateral cord of branchial plexus
79. ชายอายุ 35 ป อดน้ําอดอาหารมาเปนเวลา 16 ชม.
เพื่อเขารับการผาตัด พบ urine specific gravity = 1.025
hormone ใดในกระแสเลือดจะเพิ่มขึ้นมากที่สุด
a.insulin b.oxytocin
c.vasopressin d.glucocorticoid
e.atrial natriuretic peptide
Basic Medical Science, March 2007
สหพันธสิตนักศึกษาแพทยแหงประเทศไทย
National Council of Thai Medical Students (NCTMS)
www.nctms.in.th
-11-
80. คนไข อยูในโรงพยาบาล อดขาวอดน้ํามา 16 ชั่วโมง
ตรวจวา specific gravity 1.025 ถามวาฮอรโมนอะไรมีผล
มากที่สุด
ก. ADH ข. cortisol
ค. insulin ง. glucagon
จ. VIP
81. หญิงไทยคู อายุ 25 ป มีไข ปวดเมื่อยตามตัวมา 3 วัน
ตอมามีแผลขึ้นที่ labia minora และ labia majora
ลักษณะเปน painful red papules, vesicles และ
coalescent ulcer ตรวจลักษณะทางจุลพยาธิวิทยา พบ
intraepithelial vesicle พบ large cytoplasmic inclusion
มี clear halo ลอรอบ และพบ multinucleated giant cell
จงใหการวินิจฉัย
1. syphillis 2. acute eczema
3. gonococcal infection
4. herpes simplex infection
5. human papilloma virus infection
82. ชายอายุ 20 ป สุขภาพแข็งแรงดีมาตลอด กิน ยา
Barbiturate เกินขนาดมา 1 ชั่วโมง ตรวจรางกายพบ RR
เทากับ 12/min ถาตรวจ arterial blood gas จะพบอะไร
ก. pH 7.5 ข. PaCO2 55 mmHg
ค. PaO2 100 mmHg ง. Bicarbonate 18 mEq/L
จ. Base excess +4 mEq/L
83. เด็กชายอายุ 15 ป อาศัยอยูในชุมชนแออัด ติดยาเสพ
ติดมา 3 ป สถาบันใดมีบทบาทมากที่สุดในการปองกัน
ปญหาสุขภาพจิตดังกลาว
A. familial institute
B. political institute
C. religious institute
D. Economic institute
E. Education institute
84. ผูปวยชายเปนโรคตับ มี ascites กิน furosemide 1
เดือน ตอมาเกิดวิงเวียนเปนตะคริว ตรวจเลือดได Na 130,
K 2.5 , pH 7.58 , pCO2 50 ความผิดปกติของสมดุลกรด
เบสเกิดจากอะไร
a- uncompensated respiratory alkalosis
b- compensated respiratory alkalosis
c- uncompensated metabolic alkalosis
d- compensated metabolic alkalosis
e- mixed respiratory and metabolic alkalosis
85. ผูปวยเจ็บเขาขางขวามา3วันขยับไมไดเจาะน้ําไขสัน
หลังมาตรวจพบwbcเกิน50,000 pmnรอยละ80ผูปวย
นาจะเปนโรคอะไร
1. osteoarthritis 2.rheumatoid arthritis
3. aspergilosis 4.tuberculoarthritis
86. หญิงอายุ 17 ป ไมมีประจําเดือน แตมี breast และ
pubic hair เจริญปกติ คลําพบกอนที่ labia majora
เคลื่อนที่ไปมาได การตรวจ ultrasound ไมพบ uterus
ความผิดปกตินี้คือ
A. testicular feminizing syndrome
B. female psudohermaphrodite
C. gonadal agenesis
D. cryptorchism E. hypospadias
87. เด็กชายอายุ 5 ป มีจ้ําเขียวงายเมื่อถูกกระแทก มี
เหงือกบวม และเลือดออกตามไรฟน ผลตรวจเลือดพบ
platelet 200,000/ cu.mm. ลักษณะทางคลินิกดังกลาว
เกิดจากความผิดปรกติใด
a. Deamination of alanine
b. Hydroxylation of praline
c. Phosphorylation of serine
d. Glycosylation of hemoglobin
e. Carboxylation of glutamic acid
Basic Medical Science, March 2007
สหพันธสิตนักศึกษาแพทยแหงประเทศไทย
National Council of Thai Medical Students (NCTMS)
www.nctms.in.th
-12-
88. ชายอายุ 35 ป ถูกยิงตามรางกายหลายแหง ตรวจพบ
PR 110/min BP 80/50 24ชม. ตอมา พบ BP 70/50-80/60
แมจะไดรับสารน้ําทดแทนและการรักษาเต็มที่ การที่ความ
ดันโลหิตของผูปวยไมกลับคืนสูปกตินาจะมาจากสาเหตุใด
a. Venous return เพิ่มขึ้น
b. Central vasoconstriction
c. Central venous pressure ต่ํา
d. End diastolic pressure ลดลง
e. Total peripheral resistance ลดลง
89. ชายอายุ 35 ป มีอาการออนแรงที่แขนขวา ตรวจ
รางกายพบ spasticity และ hyperreflexia ของขาขวา แต
มี fasciculation และ hyporeflexia ของแขนขวา อยาด
ทราบวาอาการทางคลินิกดังกลาวเกิดจากพยาธิสภาพที่ใด
a. Peripheral nerve b. Ventral root
c. Spinal cord d. Brain stem
e. Alpha motor neuron
90. ผูปวยชายไดรับการวินิจฉัยวาเปนไตวายเรื้อรัง Hb 8
HCT 24% platelet 200,000 WBC 6,000 creatinin 5
mg/dl ภาวะดังกลาวเกิดจากอะไรลดลง
a. Folic acid
b. Cyanocobalamin
c. Erythropoietin
d. Thyroid hormone
e. Colony stimulating factor
91.ในการทํา systematic review เปรียบเทียบการใช
amoxicillin กับ co-amoxiclav ในการรักษา sinusitis เพื่อ
สนับสนุน การพิสูจน causal association ในขอใด
a.temporalily b.reversibility
c.consistency d.dose-effect
e.biological plausibility
92. ผูปวยโรคแผลในกระเพาะอาหาร มีการหลั่งกรด
จํานวนมากไมสามารถควบคุมดวยยาไดจึงทํา vagotomy
เพื่อลดการหลั่งกรด vagotomy สงผลตอ parietal cell
อยางไร
a. Cl-
channel b. H+
channel
c. nicotinic receptor d. muscarinic receptor
e. histamine receptor
93. ชายอายุ 30 ป ถูกงูกัดที่สนเทาซาย บวมเล็กนอย ไม
เห็นตัวงู ตอมามีเลือดออกตามไรฟน และไตวายเฉียบพลัน
ผูปวยรายนี้นาจะโดนงูชนิดใด
1. งูเหา 2. งูกะปะ
3. งูแมวเซา 4. งูทับสมิงคลา
5. งูเขียวหางไหม
94.ตรวจ ECG ของนักศึกษาแพทยคนหนึ่ง ผลในขอใด
สามารถเกิดขึ้นไดปรกติ
1. PR interval ไมเทากัน
2. RR interval ไมเทากัน
3. บางชวง P wave หายไปได
4. P wave รูปรางไมเหมือนกัน
5. QRS complex กวางขึ้น หรือแคบลง
95. เด็กไมถาย meconium หลังคลอด ตอมาทองผูกและ
ถายเหลวเปนบางครั้ง ตรวจพบไมมี ganglion cell ที่
submucosal layer เกิดจากความผิดปกติอะไร
1. migration
2. apoptosis
3. mutation
4. differentialtion
5. proliferation
Basic Medical Science, March 2007
สหพันธสิตนักศึกษาแพทยแหงประเทศไทย
National Council of Thai Medical Students (NCTMS)
www.nctms.in.th
-13-
96. เด็กทารกเพศชาย ถุงอัณฑะขางขวาใหญกวาขางซาย
ลักษณะเปนถุงน้ําใส คลําเปนแนวเดียวกับลูกอัณฑะ คาด
วาเปน
a. hydrocele
b. epispadias
c. hypospadias
d. inguinal hernia
97. squarmous cell CA ของ cervix เกี่ยวของกับเชื้อใด
มากที่สุด
a. Parvovirus b. Herpes
c. EBV d. CMV
e. HPV
98. ชาย 50 ป เหนื่อยงาย น้ําหนักลด x-ray พบกอน 3 cm.
ที่ apex ของปอดขวา ผาตัดกอนออกมา ดู histo พบดัง
ภาพ คาดวาผูปวยเปน
a. Aspergillosis
b. Tuberculosis
c. Lung abscess
d. Cryptococcosis
d. Bronchial carcinoma
99. เด็กชายอายุ 5 ป มีไขสูง ซึม อาเจียน ตรวจพบ
stiffness of neck ผลการตรวจ CSF พบ WBC 1,200
cell/cu.mm. N 90% MN 10% protein 150 μg/dl sugar
20 μg/dl(blood sugar 100) เชื้อกอโรคใดมีโอกาสเปนไป
ไดมากที่สุด
a. Enterovirus
b. Neagleria floweli
c. Neisseria meningitidis
d. Cryptococcus neoformans
e. Mycobacterium tuberculosis
100. เด็กชายอายุ 7 ป 3 สัปดาหกอนมีไข 2 วันกอนเขา
โรงพยาบาล มีอาการเทาบวม กดไมบุม ตรวจรางกายพบ
BP170/100 protein 2+ WBC 10-20/HPF พบ
numerous dysmorphic RBC พบ RBC cast 1-2/HPF
อยากทราบวามี pathology ที่บริเวณใด
a. Glomerulus
b. Proximal tubule
c. Loop diuretic
d. Distal tubule
e. Interstitium
101. ชายอายุ 35 ป ไมเคยออกกําลังกายมากอน พอเตน
แอโรบิกครั้งแรก เสียเหงื่อเปนจํานวนมาก ตรวจ BP กอน
และหลังปรกติ อยากทราบวารางกายใชระบบ renin-
angiotensin system ในการรักษา BP อยางไร
a. Vasodilation
b. ↑ renal blood flow
c. ↑ sodium reabsorption
d. ↑ GFR
e. ↓ angiotensin
102. ผูหญิงวัยกลางคน เปนไขต่ํามา 2 วัน แลวปวดหลัง
บริเวณลางซาย แลวลามมาที่ขาหนีบซายอยางฉับพลัน
x-ray พบ ureteral calculi นาจะเกี่ยวกับการติดเชื้อใด
a. Esterichia coli
b. Protius mirabilis
c. Enterococus faecialis
d. Krebsiella pneumonia
e. Staphylococcus saprophyticus
Basic Medical Science, March 2007
สหพันธสิตนักศึกษาแพทยแหงประเทศไทย
National Council of Thai Medical Students (NCTMS)
www.nctms.in.th
-14-
103. ชายอายุ 70 ป สูบบุหรี่จัดมา 40 ป เหนื่อยงาย หอบ
เปนครั้งคราว ตรวจ x-ray พบวาเปน emphysema คาใด
ของ lung function test ลดลง
a. Airway resistance
b. Lung compliance
c. Intrapleural pressure
d. FVC
e. FEV1
104. ยา X ให ผลเปน ½ เทาของ max ที่ 5 mgใน 1 hr
ยา Y ให ผลเปน ½ เทาของ max ที่ 25 mgใน 2 ชั่วโมง
ยา X มีผลเปนกี่เทาของยา Y
a. 5 b. 10
c. 25 d. 40
e. 50
105. หญิงอายุ 35 ป ตั้งครรภ 32 สัปดาห ตรวจ
ultrasonography พบวาเด็กในครรภมีไตซายขางเดียว
และมีปริมาณน้ําคร่ํานอย ความผิดปรกติดังกลาวนาจะ
เกิดจากสาเหตุใด
a. Oligohydramnios
b. Polycystic kidney
c. Ureteric bud duplication
d. Degeneration of mesonephros
e. Absence of metanephrogenic tissue
106. เด็กชายอายุ 5 ป มีจ้ําเขียวงายเมื่อถูกกระแทก มี
เหงือกบวม และเลือดออกตามไรฟน ผลตรวจเลือดพบ
platelet 200,000/ cu.mm. ลักษณะทางคลินิกดังกลาว
เกิดจากความผิดปรกติใด
1. Deamination of alanine
2. Hydroxylation of praline
3. Phosphorylation of serine
4. Glycosylation of hemoglobin
5. Carboxylation of glutamic acid
107. (A).รูป. ถามวาเปนกระบวนการใด { normal cell
→chromatin dense → nucleus แตกเปน bleb →
cell อื่นมาเก็บกิน }
a. necrosis b. karyolysis
c. apoptosis d. pinocytosis
e. endocytosis
108. การสํารวจ CA breast stage 1,2,3,4 พบผูปวยรอย
ละ 20,40,60,80 ตัวแปรการศึกษานี้เปนแบบใด (ขอนี้มี 4
ตัวเลือก)
a. Ratio
b. Interval
c. Nominal
d. Ordinal
109. ชายอายุ 20 ป ขณะเลนฟุตบอลหายใจเร็วและแรง
ขึ้น การเพิ่มของปจจัยใดทําใหรางกายไดรับอากาศใหม
มากที่สุดขณะเลนฟุตบอล
a. Tidal volumn
b. RR
c. Minute ventilation
d. Dead space ventilation
e. Inspire reserve volumn(IRV)
120. ชาย 30 ป ไข ไอ หอบ เหนื่อย 2 วัน ตรวจรางกายพบ
fine crepitation ที่ปอดลางขวา แสดงถึงการอักเสบ
บริเวณใด
a. Alveoli
b. Trachea
c. Main bronchus
d. Lobar bronchus
e. Segmented bronchus
Basic Medical Science, March 2007
สหพันธสิตนักศึกษาแพทยแหงประเทศไทย
National Council of Thai Medical Students (NCTMS)
www.nctms.in.th
-15-
121. ชาย 65 ป เปน COPD+bronchietasis มีประวัติสูบ
บุหรี่เยอะ เสมหะมาก หายใจเหนื่อยแมพัก คาทางสรีระ
วิทยาใดสัมพันธกับผูปวย
a. FEV1/FVC ปรกติ
b. Airways resistance สูงขึ้น
c. Residual volumn ลดลง
d. Lung compliance ลดลง
e. Lung capacity เพิ่มขึ้น
122. หญิง 30 ป ทานอาหารทะเล เกิดผื่น erythrematous
plaque cell ใดเปน mediator หลัก
a. Basophil
b. Eosinophil
c. Macrophage
d. Neutrophil
e. Lymphocyte
123.ชายอายุ 30ป เปนthalassemiaตองไดรับเลือดแตเด็ก
มีผิวคล้ํา serum glucose 140 พยาธิสภาพของตับออน
เปนอยางไร
a. Insulinitis
b. Pancreatitis
c. มี amyloid สะสม
d. Hemochronatosis
124. ชาย 60 ป เปนเบาหวานมา 20 ป มีแผลที่เทา ฉีด
Stem cell แลวพบวาแผลหายและเล็กลง stem cell คือ
อะไร
A. plasticity
B. maturation
C. induction
D. migration
E. cell differentiate
125. เด็กชายอายุ 10 ขวบ มีไข 2 วัน ตัวเหลือง ซีด
Hct 25% Hb 7 WBC 5000 ( N 65% L 35% M 5% )
Plt 250000 ตรวจพบHeinzbodyเกิดจากกกระบวนการใด
a. Precipitate ของ Ribosome
b. Precipitate ของ Hemoglobin
c. Iron inclusion
d. การแตกของ Nucleus
e. Aggregation ของ ER
126. เพศชาย 70 ป ออนเพลียมากมา 7 วัน มีประวัติ
ปวดทองและอุจจาระดํามา 3 เดือน Hct 23 WBC
3200 ( N 30% L 70% ) Plt 70000 นาจะเกิดจาก
สาเหตุใด
a. เสียเลือดเรือรัง
b. Autoantibody
c. ขาดเซลลตั้งตนในไขกระดูก
d. มีการใชเกร็ดเลือดเปนจํานวนมาก
e. ขาดสารอาหารในการสราง RBC
127. เด็กมีผิวกายซีด สีผมจาง ขนออนเปนสีขาว ตาดํา
เปนสีแดง มีพอและแมปกติ นาจะเกิดจากความผิดปกติ
ของอะไร
a. ectoderm
b. mesoderm
c. endoderm
d. neural crest
e .neural tube
128. หญิงแตงงานมา 3 เดือนหลังจากนั้น 1 เดือน
ประจําเดือนหายไป ควรตรวจ hormoneใดเพื่อดูการ
ตั้งครรภ
a. estrogen b. progesterone
c. FSH d. LH
e. HCG
Basic Medical Science, March 2007
สหพันธสิตนักศึกษาแพทยแหงประเทศไทย
National Council of Thai Medical Students (NCTMS)
www.nctms.in.th
-16-
129. pt หญิงอายุ 40 ปมีกอนที่คอ ตอมไทรอยด PR 70
ไมมีอาการใจสั่น มือสั่น พบขุยสีขาวที่ผิวกอน
นาจะเปนอะไร
a. follicular adenoma
b. papillary carcinoma
c. follicular carcinoma
d. medulary carcinoma
e. anaplastic carcinoma
130. N-acetylcysteinที่ใชในการแกไขภาวะ
paracetalmol toxicity มีกลไกในอารออกฤทธิ์เกี่ยวของ
กับกลไกใด ของ liver
a. N-acetylation
b. N-hydroxylation
c. sulfer conjugate
d. glutathione conjugate
e. glucolonide conjugate
131.หญิงอายุ 30ตั้งครรภแรกตรวจเลือดพบ Anti-HBsAg
ได positive สวน Anti-HBcAb และ Anti-HBsAb ได
negative 6 เดือนตอมา ตรวจAnti-HBsAg และ Anti-
HBcAb ได positive สวน Anti-HBsAbได negative
จงแปลผลตรวจ Serology
a. เคยไดรับวัคซีน
b. อยูในชวงพาหะ
c. อยูในชวงระยะฟกตัว
d. ติดเชื้ออักเสบเฉียบพลัน
e. เคยไดรับเชื้อและมีภูมิตานทานอยูแลว
132. Patient มีอาการแนนหนาอกดานซายมา 1 สัปดาห
ตรวจรางกายพบ dullness, breath sound ขางซายนอย
กวาขางขวา และมี trachea เบนไปทางขวา นาจะเปนโรค
ใด
a. pneumothorax
b. pleural effusion
c. lobar pneumonia
d. lung atelectasis
e. rib fracture
133. ชายอายุ 20 มีอาการอาเจียน ปวดทองใตชายโครง
ดานขวา ราวไปถึงตนแขนขวา ไดรับการวินิจฉัยวาเปน
acute hepatitis ถามวา การตรวจในขอใดบงชี้วา มีอาการ
รุนแรงที่สุด
a. Ammonia > 80
b. AST(SGOT) > 2000
c. Blood sugar > 160
d. Prothrombin > 38
e. Alkaline phosphatase > 300
134. ผูชาย 50 ป เหนื่อยหอบ หนาบวม มี engorge
jugular venous , dilate superficial vein on chest มี
hilar lymph node โต ไมมีรอยโรคอื่นๆ จากการ X-ray
นาจะเปนโรคใด
a. Tuberculosis
b. Aspergilosis
c. Lung abcess
d. Brocheal adenoma
e. Brochogenic CA
Basic Medical Science, March 2007
สหพันธสิตนักศึกษาแพทยแหงประเทศไทย
National Council of Thai Medical Students (NCTMS)
www.nctms.in.th
-17-
135. เด็กชายอายุ 5 ป ไขสูงมา3วัน ชัก ซึม Stifness of
neck ตรวจรางกายพบวาไมรูสึกตัว ตรวจ CFS โดยการ
ยอม Gram พบ Gram negative diplococci ควรใหการ
ดูแลใกลชิดผูปวยยังไง
a.ใหยาปฏิชีวนะทุกคน
b.ใหยาปฏิชีวนะทุกคน และใหวัคซีนเพิ่มกับญาติ
ผูปวย
c.ใหยาปฏิชีวนะทุกคน และใหวัคซีนเพิ่มกับ บุคลากร
ทางการแพทย
d.ใหวัคซีนกับทุกคน และใหยาปฏิชีวนะกับญาติ
ผูปวย
e.ใหวัคซีนกับทุกคน และใหยาปฏิชีวนะกับบุคลากร
ทางการแพทย
136.ผูชายประสบอุบัติเหตุทางรถยนตเปนผลใหมีการ
เปลี่ยนแปลง RR (เพิ่มขึ้น), BP 200/100 BPเพิ่มขึ้นเพราะ
เหตุใด
a. Preload เพิ่ม b. afterloadเพิ่ม
c. Sympathetictone ลด
d. มี vasoconstriction of peripheral vessels
137. ผูหญิงอายุ 60 ปเปน เบาหวานมี BMI=30 ถามวา
ควรจะใหยาใดในการรักษาผูปวยคนนี้
a.Metformin b.Acarbose
c.Meglitinide d.Glitazone
e.Glipizide
138. คนขึ้นที่สูง 800 ฟุตเหนือน้ําทะเล มีการหายใจเร็วขึ้น
เกิดจากการกระตุนกลไกใด
a. J-receptor
b. Irritant receptor
c. peripheral chemoreceptor
d. central chemoreceptor
139. Cushing syndrome จากยาหมอ ตรวจเลือดจะพบ
อะไร
a. Na+
นอย b. K+
นอย
c. Ca+
มาก d. Cl-
มาก
e. metabolic alkalosis
140. สารใดยับยั้ง apoptosis
A. P53 protein B. bcl-2
C. IL-6 D. tyrosine kinase
E. TNF
141. ชายอายุ 40ป ปวดที่ first metatarsophalangeal
joint , serum uric acid สูง ขอใดผิด
a. เปนโรค gout
b. เกิดจากความผิดปกติใน pyrimidine salvage
pathway
c. กินอาหารที่มี nucleic acid สูง
d. มีภาวะ acidosis สูง
142.เด็กอายุ 10 ป ลมแลวมี fracture ที่ humorus ถามวา
มีผลขางเคียงตอ เสนประสาทใด
A. ulna nerve
B. median nerve
C. radial nerve
D. Internal cord of brachial plexus
E. External cord of brachial plexus
143. ผูปวยไตวายเรื้อรัง K= 6, Na = 135 , Cl ปกติ ถาม
วาควรใหยาอะไร
1. Spinoloractone 2. Priqnerene
3. Amiloride 4. Furosemide
5. Eplereone
Basic Medical Science, March 2007
สหพันธสิตนักศึกษาแพทยแหงประเทศไทย
National Council of Thai Medical Students (NCTMS)
www.nctms.in.th
-18-
144. ชายอายุ ? ป มีอาการ weak ที่ distal limb แลว
อาการ progress ที่ proximal เจาะ CSF ได wbc 3,
protein 250, sugar 80, เขาไดกับโรคใด
A. Toxic neuropathy B. Myasthenia gravis
C. Reverse
D. Guillain-Barr syndrome
C. Brown-sequard
145. ผูปวนเดินเกร็ง หนาไรอารมณ มีresting tremor
Metabolism สวนใดของระบบประสาทที่นอยที่สุด
A. Cerebellum B. Hippocampus
C. basal ganglia D. anterior horn cell
E. mammillary body
146. ชายอายุ 35 ป มีอาการออนแรงที่แขนขวา ตรวจ
รางกายพบ spasticity และ hyperreflexia ของขาขวา แต
มี fasciculation และ hyporeflexia ของแขนขวา อาการ
ทางคลินิกดังกลาว นาจะเกิดจากพยาธิสภาพที่ใด
ก. peripheral nerve ข. ventral root
ค. spinal cord ง. brain stem
จ. alpha motor neuron
147. หญิง อายุ 20 ป เปน จิตเภท รักษาดวย Haloperidol
แลวมีอาการลิ้นแข็ง เกิดจากกลไกใด
A. inhibit dopaminergic B. inhibit cholinergic
C. inhibit serotonin
D. กระตุน dopaminergic
E. กระตุน cholinergic
148. ผูปวยเปนโรค B-thalassemia  HbE disease
แตงงานกับคนที่เปน HbE homozygote โอกาสมีลูกเปน
โรค thalassemia เทากับขอใด
A. 1 B. 3/4
C. 1/2 D. 1/4
E. 0
149. ผูหญิงแก มีอาการหลงลืม แพทย Dx Alzheimer มี
ความผิดปกติทีสมอง สวนใด
A. crus cerebri B. cerebellum
C. hippocampus D. Globus pallidus
150. ทารกชายอายุ 1 วัน มีอาการหายใจลําบาก ตรวจ
รางกายพบหนาทองแฟบ ฟงไดเสียง peristaltic sound ที่
ชองอกดานขวา หัวใจและtrachea ถูกดันมาอยูทางดาน
ขวา ลักษณะทางคลินิกดังกลวเกิดจากความผิดปกติของ
โครงสรางใด
1. later body wall
2. septum tranversum
3. costodiaphragmatic recess
4. pluroperitoneal membrane
5. dorsal mesentery of esophagus
151. ชายอายุ 40 ป ไดรับยาเคมีบําบัดรักษามะเร็งเม็ด
เลือดขาว หลังจากนั้นมีไข จํานวนเม็ดเลือดขาวลดลง
ไดรับยาปฏิชีวนะเปนเวลานาน ระหวางที่ไดรับยา
ปฏิชีวนะมีอาการปวดทอง อุจจาระรวง ไมพบเม็ดเลือด
แดงและ เม็ดเลือดขาว ผลการเพาะเชื้อจากอุจจาระไมพบ
normal flora เชื้อที่เปนสาเหตุนาจะเปนเชื้อใด
1. Clostridium difficile
2. Shigella dysenteriae
3. Salmonella enterica
4. Yernisia enterocolita
5. Plesiomonas shigelloides
152. Pathway ที่ใหพลังงานแก RBC
a. Urea cycle
b. Kreb’s Cycle
c. Beta oxidation
d. Lactose fermentation
e. Glycolysis
Basic Medical Science, March 2007
สหพันธสิตนักศึกษาแพทยแหงประเทศไทย
National Council of Thai Medical Students (NCTMS)
www.nctms.in.th
-19-
153. ผูปวยหญิงอายุ 45 ป เพลียซีด อาเจียนมาก และ
บวมมา 2 สัปดาห เมื่อ 2 ป กอนเคยมีอาการบวม ทั้งตัว
ตรวจรางกายพบ BP 180/110 ,lung sound: crepitation
ที่ปอดสวนลาง บวมที่หนาขาทั้ง 2 ขาง urinalysis พบ
specific gravity 1.01 albumin 2+ wbc 10/lpf rbc
20/lpf broad waxy cast 1-2 hpf คนไขนาจะเปน
a. Chronic renal failure
b. Renal cell carcinoma
c. Acute tubular necrosis
d. Renal tubular acidosis
e. Rapid progressive glomerulus nephritis
154. ผูปวยชายอายุ 60 ป มีอาการตัวเหลืองตาเหลือง คัน
ทั่วตัว และอุจจาระซีด ตรวจรางกายคลําตับได 1 ซม. ใต
ชายโครงขวา ผูปวยนาจะมีระดับของสารใดสูงขึ้น
a. stercobillin b. serum albumin
c. urine urobillinogen d. plasma prothrombin
e. serum direct bilirubin
155. ผูปวยหญิงโสดอายุ 30 ป ไมมีประจําเดือนมานาน
มีน้ํานมตลอด 8 เดือน ที่ผานมาตรวจ urine pregnancy
test ไดผลลบ ผูปวยนาจะมีระดับสารใดในเลือดสูง
a.prolactin b.estradiol
c.progesterone d.leutinizing hormone
e.follicular stimulating hormone
156. หญิงมีกอนที่ขอเทา แผลมีรูเปดหลายรู บีบแลวมี
หนองปนเม็ดเล็กๆ สีเหลือง ยอม modified acid fast
ใหผลเปนลบ
1. Nocardia asteroids
2. Penicillium manesfii
3. Clostidium perfringen
4. Mycobacterium chelone
5. Streptomyces spp
157. หญิงหาบเร อายุ 30 ป ไมมีโรคประจําตัว แตงงาน
มา 5 ป มาพบแพทยดวยเรื่องไมมีบุตร มีประวัติปสสาวะ
แสบขัดบอย และตกขาว ซื้อยากินเอง สามีมีประวัติเที่ยว
หญิงบริการบอยโดยไมสวมถุงยางอนามัย ผลการตรวจ
รางกายอสุจิของสามี analysis อยูในเกณฑปกติ ปจจัยใด
ที่ทําใหผูหญิงคนนี้ไมมีบุตร
a. อาชีพ
b. เศรษฐกิจ
c. ซื้อยากินเอง
d. สามีสําสอน
e. มีโรคติดตอทางเพศสัมพันธบอย
158. หญิง 25 ป ตั้งครรภ 40 สัปดาห เจ็บทองพรอมคลอด
แตไมมีแรงเบงลูก จะใหยาชนิดใดจะไดคลอดบุตรงาย
a. Oxytocin b. Ritrodine
c. Ergotarmine d. Vasopressin
e. Bromocriptine
159. ชายไทยอายุ 50 ป มีอาการเจ็บหนาอก แพทยให
การวินิจฉัยวาเปน acute myocardial infarct จึงใหยา
ละลายเลือดเพื่อรักษา ถามวายานี้มี mechanism เพิ่ม
activity ของสารใด
a. plasmin b. Protein C
c. AntiThrombin d. Prostacyclin
e. Phospholipid
160. กลไกใดของเกล็ดเลือดเกิดเปนอันดับแรกใน
กระบวนการหามเลือด
a. เกิดการหลั่ง ADP
b. มีการปลอย Prostacycline
c. Activation ของ COX enz
d. Activation ของ thrombomodulin
e. Activation ของ glycoprotein และ VWF
Basic Medical Science, March 2007
สหพันธสิตนักศึกษาแพทยแหงประเทศไทย
National Council of Thai Medical Students (NCTMS)
www.nctms.in.th
-20-
161. หญิงอายุ 20 ป กินยานอนหลับไมทราบขนาด ไมมี
สติ (unconscious) หายใจชาและลึก ประมาณ 8 ครั้ง/
นาที วัด arterial blood gas : pH = 7.07 paO2 = 32
paCO2 = 88 HCO=24 แพทยไดใชเครื่องชวยหายใจ
เครื่องชวยหายใจมีประโยชนตอคนไขรายนี้ในดานใดมาก
ที่สุด
a. เพิ่ม oxygen ในเลือด
b. เพิ่ม minute ventilation
c. เพิ่ม diffusing capacity
d. ลด resistance
e. ลด intrapulmonary shunt
162. ชายอายุ 40 ป อาชีพแกะสลักชื่อบนหินออน ไอและ
หอบเหนื่อยมา 2 ป ประวัติสัมพันธสารพิษ มา 4 ป พยาธิ
สภาพที่เกิดขึ้นจะเกิดขึ้นที่ใด
a. Nose
b. nasopharynx
c. Trachea
d.bronchus
e. alveoli
163. ชายอายุ 15 ป มีกลิ่นตัวทําใหขาดความมั่นใจ
ปญหาดังกลาวเกิดจากโครงสรางใดของผิวหนัง
ก. epidermis
ข. hair follicle
ค. apocrine gland
ง. sebaceous gland
จ. arrector pilli muscle
164. การศึกษาเปรียบเทียบประวัติการไดรับวัคซีนปองกัน
โรคหัดในวัยเด็กระหวางนักเรียนที่เปนโรคและไมเปนโรค
หัด พบวา odds ratio ของการเปนโรคหัด ในกลุมที่ไดรับ
วัคซีนเปรียบเทียบกับกลุมที่ไมเคยไดรับวัคซีน = 0.05
165. ประสิทธิผลของวัคซีนจากการศึกษาดังกลาว = รอย
ละเทาไร
a. 0.05 b. 0.95
c. 5 d. 95
e. 99.95
166. ครูพบนักศึกษาชายหญิงกอดจูบกันที่ปายรถเมล จึง
เขาไปตักเตือนวาเปนพฤติกรรมที่ไมเหมาะสม ครูใชเกณฑ
ใดเปนเครื่องมือในการตัดสินพฤติกรรมนักเรียน
a. faith b. value
c. norms d. belief
e. attitude
167. หญิงอายุ 50 ป มีอาการเหนื่อยงายมา 1 เดือน มี
อาการชามือชาเทา มีประวัติรับประทานอาหารมังสวิรัติมา
2 ป ตรวจรางกายพบวาซีด จากอาการทางคลินิก แสดงวา
ขาดวิตามินใด
a. Retinol b. Thiamine
c. Riboflavin d. Cobalamine
e. Niacin
168. ชายอายุ 60 ป มีอาการอุจจาระรวงสลับทองผูกมา 3
week adenocarcinoma sigmoid colon แพทยไดทํา
รักษา โดยการผาตัดสวนที่เปนมะเร็งออกและทําเคมีบําบัด
แลว follow up ดวยการดู tumor marker ชนิดใด
a. CA-125 b. CA 15-3
c. alpha fetoprotein
d. lactic dehydrogenase
e. Carcinoembriogenic antigen
169. เด็กอายุ 1 ป มีความบกพรองของ hydrolytic enz. มี
ผลตอ organelle ใด
a. acrosome b. lysosome
c. ribosome d. microsome
e. peroxisome
Basic Medical Science, March 2007
สหพันธสิตนักศึกษาแพทยแหงประเทศไทย
National Council of Thai Medical Students (NCTMS)
www.nctms.in.th
-21-
170. อัตราการไหลเวียนเลือด 100 ml/min ถาหลอดเลือด
มีรัศมีลดลง 50% โดยความดันเทาเดิม จะมีอัตรา
ไหลเวียนเลือดเทาไร
a. 6.25 b. 25
c. 50 d..... e.....
171. เด็ก 8 ป ลักษณะเตี้ยขาโกง Ca 7.5 mg/dl,
phosphate 2.8mg/dl, alkaline phosphatase
300microI/L อาการของเด็กเกิดจากการขาดสารอะไร
a. Ca b. Vit D
c. Vit C d. Calcitonin
e. PTH
172. เด็กอายุ10ป มีอาการอานเพลีย occult blood
positive ไดทําstool exam ผลปรากฏดังภาพ
ควรแนะนําใหผูปวย
a. ไมเดินเทาเปลา
b. ไมกินผักสด
c. ไมกินปลาราดิบ
d. ไมกินหมูสุกดิบ
e. ลางมือทุกครั้งหลังออกจากหองน้ํา
173. หญิงอายุ50ป เลือดกะปริบกะปรอยทางชองคลอดมา
1 week ตรวจภายในพบมดลูกโตขึ้นเล็กนอย จากการ
ตรวจชิ้นเนื้อพบ proliferative ของendometrial gland อยู
ชิดกันหนาแนน, cellบุผนังมดลูกมี Nucleus ใหญ, ติดสี
เขม, Nucleolusชัดเจน พบ mitotic figure all
ปรากฏการณที่พบเกิดจากอะไร
1. endometriosis
2. HPV infection
3. Gonococcal cervicitis
4. Infertility
5. Low-socio-economic status
174. หญิงอายุ 40 ป ทําหนาที่เปนเจาหนาที่ฝายบัญชีมีคา
BMI 26.5 2 ปที่แลวไดรับการผาตัดรังไขออกทั้ง 2 ขาง
ตอนนี้เปน Osteoporosis อยูถามวา อะไรที่เปนปจจัยที่ทํา
ใหคนนี้เปน osteoporosis
a. อายุ
b. เพศ
c. ดัชนีมวลการ
d. อาชีพ
e. ผาตัด ovary ออก
175. ชาย 45 ป ปวดเปนๆหายๆ ที่ MTP jt. ของนิ้วหัวแม
เทาขางขวา uric 8.2 mg/dl ผิดปกติ metabolism ของ
สารใด
a. purine
b. protein
c. glucose
d. cholesterol
e. pyrimidine
176. ชาย 40 ป มีอาการปวดกลามเนื้อ กลามเนื้อคลายตัว
ชา ขาดสารอะไร
a. Ca
b. phosphate
c. Creatinine phosphate
d. ADP
e. ATP
Basic Medical Science, March 2007
สหพันธสิตนักศึกษาแพทยแหงประเทศไทย
National Council of Thai Medical Students (NCTMS)
www.nctms.in.th
-22-
177. หญิงอายุ 30 ป ญาติพามาปรึกษาเนื่องจากเปลี่ยน
งานบอย ใหเหตุผลวาเพื่อนรวมงานและหัวหนาคอยแกลง
อยูในหองตลอดเวลา บางครั้งพูดคนเดียว โมโหญาติคิดวา
ญาติคอยแกลงเปนอาการใด
a. delirium
b. schizophrenia
c. delusional disorder
d. Major depressional disorder
e. Substance related disorder
178. เด็กหญิงอายุ 2 ป มีกระจุกบน (hair patch) บริเวณ
sacral region ตรวจรางกาย ไมพบความผิดปกติในระบบ
ประสาท อาการแสดงนี้มักพบรวมกับอาการใด
1. Rachischisis 2. Spina bifida cystica
3. Spina bufida occulta
4. Spina bifida with myeloschisis
5. Spina bifida with meningomyelocele
179. อายุ27ป กิน carbamet แลวมีอาการปวดทอง,
muscle fasciculation, น้ําลายยืด, เหงื่อออก, PR 60
beat/min หมอให Atropine เปนยารักษาแลวจะยังคงมี
อาการใดคงอยู
1. ปวดทอง 2. Muscle fasciculation
3. น้ําลายยืด 4. เหงื่อออก
5. 60 beat/min
180. ชายอายุ 20 ปสุขภาพแข็งแรง หลังจากบริจาคเลือด
ไป 1 unit เกือบหมดสติเปนลม BP 80/50 , PR 50/min
นาจะเกิดจากสาเหตุใด
a. cardiogenic shock
b. hypovolumic shock
c. vasovagal attack
d. orthostatic hypotention
e. stroke adams
181. ชายอายุ 50 ป มีแผลที่หลัง เปนผื่นขาววงกลม ขอบ
แดงนูน ขูดบริเวณแผลมาตรวจ KOH preparation พบเชื้อ
hyaline septate hyphae ใชยาใด
a. Fluconazole b. Clotimazole
c. Amphotericin B d. Bemzyl benzoate
e. Sodium thiosulfate
182. ลําดับเบสของผูปวยที่มี mutatuion เปน
..ATGCCAAA.. หากตองการหาDNA probe ควรใชลําดับ
เบสใด
ก. UUUGGCAU
ข. UACGGUUU
ค. TTTGGCAT
ง. TACGGTTT
จ. TTTGGUAT
183. ในการทดลองยา X ที่มีผลตอ telomerase เพื่อเปน
ยาตานCA ยาควรยับยั้งกระบวนการใดของ CA
ก. DNA stability
ข. DNA replication
ค. DNA degradation
ง. Gene rearrangement
จ. Spindle fiber attachment
184. หญิงอายุ 24ป อยุระหวางใหนมบุตร พบวามีกอนที
RT outer quadrant ที่เตานมขวา กดเจ็บ รอบๆกอนรอน
และมีสีแดง กินยาแลวอาการดีขึ้น ตอมาบริเวณที่เคยเปน
กอนกลายเปนผิวหนังที่บุมลงไป สิ่งที่เกิดขึ้นเปนพยาธิ
สภาพแบบใด
a. fibroadenoma
b. fibrocystic change
c. traumatic fat necrosis
d. breast abscess with scar
e. infiltrating mammary adenocarcinoma
Basic Medical Science, March 2007
สหพันธสิตนักศึกษาแพทยแหงประเทศไทย
National Council of Thai Medical Students (NCTMS)
www.nctms.in.th
-23-
185. ผูปวยหญิงอายุ 35 ป ตัดไขและปกมดลูกออกทั้งสอง
ขางเนื่องจากมีเนื้องอก แตผูปวยกลัววาจะมีภาวะกระดูก
พรุน ถามวายาหรือสารใดที่เปน single treatment ของ
กระดูกพรุน
1. Ca 2. Vit D
3. Prednisolone 4. Progesterone
5. Estrogen
186. เด็กผูหญิงอายุ 6 เดือน แมพบวามีน้ําซึมออกมาจาก
สะดือ นาจะเปนโรคใด
ก. Urachal fistula
ข. Omphalocele
ค. gastrichisis
ง. neuroblastoma
187. คนไขประสบอุบัติเหตุ ทางรถยนต มีปสสาวะออกมา
5000 ml urine graviy 1.015 คนไขคนนี้มีความผิดปกติที่
ใด
ก. …….
ข. การ reabsorb sugar
ค. การขับ...
ง. การ reabsorb water ที่ collecting duct
จ. การ reabsorb Na ที่ PCT
188. หญิงอายุ 50 ป ทานขาวมันไก 2 จาน และฟกทอง
แกงบวช 1 ถวย หลังจากนั้นรูสึกทองอืด และหลังจากนั้นก็
รูสึกอิ่ม ถามวา hormone จาก GI อะไรที่ทําใหเกิด
เหตุการณนี้
ก. Motilin
ข. Gastrin
ค. Secretin
ง. CCK
จ. VIP
189. หญิงอายุ 30 ป อุจจาระรวงเปนน้ําสลับกับอาเจียน
เปนเวลา 6 ชั่วโมง กอนหนานี้ทานขาวคลุกกะป คิดวาเชื้อ
อะไรเปนสาเหตู
A. E.coli
B. Virio chlorella
C. Shigella
D. Bacillus cereus
E. Aeromonas hydrophilia
190. สามีภรรยา มีลูกเปน Hb H เจาะเลือดสามีภรรยา
Hct Hb MCV Reticulocyte
สามี 38 13 72 1%
ภรรยา 39 13 87 0.5%
Genotype ใดถูกตองที่สุด
a. สามี αα/--
b. ภรรยา α-/α-
c. ภรรยา αα/αα
d. สามี βoβt
e. ภรรยา βoβo
191. เด็กเปน giloma ไดรับรังสีรักษา ตอมากอนไดยุบลง
นิดเดียว เปนเพราะอะไร
a. inactivation p53
b. inactivation BCl-2
c. Inactivation MPL-
d. Activation retinoblastoma
e. Activation BCL
192. กินมังสวิรัต 2 ป ขาด Vit อะไร
1. thiamine 2. folate
3. Riboflavin
4. B12
5. niacin
Basic Medical Science, March 2007
สหพันธสิตนักศึกษาแพทยแหงประเทศไทย
National Council of Thai Medical Students (NCTMS)
www.nctms.in.th
-24-
193. ผูปวยชายอายุ 30 ป มีประวัติปวดทองและถายดํา
มา 3 เดือน คลําไมพบตับ มาม ตอมน้ําเหลืองโต ตรวจ
พบ Hct 23% WBC 3,200 Neu 30% Lymphocyte 70%
Platelet 50,000 MCV 89 ขอใดอธิบยภาวะโรคของผูปวย
ไดถูกตอง
1. การสราง RBC นอยลง
2. มามทําลาย RBC มากไป
3. มี Ab มาทําลาย RBC มาก
4. ขาดเซลลตนกําเนิดในการสราง
5. ขาดสารอาหารในการสราง
194. เด็กชายอายุ 5 ป เปน Hemophilia A ใชอะไรในการ
ชดเชย coagulation
1. Aged plasma
2. Cryoprecipitate
3. Fresh frozen plasma
4. Platelet
5. Fresh whole blood
195. ศูนยมะเร็งแหงหนึ่งศึกษาเกี่ยวกับความรุนแรงของ
มะเร็งเตานม มี 4 ระยะคิดเปอรเซ็นต ดังนี้ 20,35,35,10
การศึกษานี้มีลักษณะขอมูลแบบใด
1. Interval 2. Ordinal
3. Ratio
4. Nominal
196. คนไขปวดทองบริเวณรอบสะดือ จากนั้นยายมาที่
ทองนอยดานขวา หมอ diag วาเปนไสติ่งอักเสบ คนไขจะ
กดเจ็บบริเวณใด
a. ¼ ของระยะทางจาก ASIS ไปสะดือ
b. ¼ ของระยะทางจาก AIIS ไปสะดือ
c. 1/3 ของระยะทางจาก ASIS ไปสะดือ
d. 1/3 ของระยะทางจาก AIIS ไปสะดือ
e. ¼ ระยะทางจาก iliac crest ไปสะดือ
197. ทารก 3 วันคลอดกอนกําหนด หนัก 2,300 g หายใจ
เร็ว ไมเขียว ฟงเสียง murmur ไดที่ left 2nd
intercostals
space เมื่อให indomethacin เสียง murmur หายไป
ทารกนี้มีโรคหัวใจคืออะไร
a. Aortic stenosis b. Pulmunic stenosis
c. ASD d. VSD
e. PDA
198. เด็กเปนฝที่ขา เมื่อ Phagocytosis เขาไป ใช enz ใด
ใน O2-dependent mechanism ใน neutrophil
1. Lysozyme 2. Defensin
3. Hydrolaze 4. Lactoferrin
199. หญิงอายุ 35 ป ซีด ออนแรง เหนื่อยงาย เปน
peptic ulcer ถายอุจจาระดํา ตรวจรางกายพบซีดปาน
กลาง มีไข เปนๆหายๆ ตาเหลือง ตับไมโต แต มามโต
ตรวจเลือดพบ Hb 8 g/dL Hct 30 Reticulocyte 8%
Platelet 250,000/mm3 MCV 86 fL ภาวะดังกลาวเกิด
จากสาเหตุใด
1. autoimmune
2. ขาด Vit B12
3. Hb ผิดปกติ
4. ขาด enzyme
5. Post hemorrhage
200. เด็กชาย 5 ป มีอาการเขาไดกับ down syndrome
ผลการตรวจ karyotype ได 46,XY ความผิดปกติใดเขาได
กับเด็กคนนี้
1.deletion
2.mosaicsm
3.translocation
4.somatic mutation
5.undetected trisomy
Basic Medical Science, March 2007
สหพันธสิตนักศึกษาแพทยแหงประเทศไทย
National Council of Thai Medical Students (NCTMS)
www.nctms.in.th
-25-
200.เด็กชายชาวเขาอายุ 1ป ไมไดรับวัคซีน ไอมา2สัปดาห
ไอมากเปนชุดๆ ไอจนตัวงอและหนาแดง ไอมาตอนกลางคืน
ไมมีไข ฟงปอดปกติ ตรวจพบ Subconjunctival
hemorrhage ถามวา อาการทาง Clinic เกิดการกลไกใด
1. การเปลี่ยนแปลง Normal flora
2. Mucosal inflammation
3. Increase mucous secretion
4. Pseudomembranous formation
5. Reduced mucociliary clearance
201. ชายอายุ 20 ป โดนงูเหากัด ไดรับเซรุมแกพิษงู มี
อาการ urlicaria rash generalized lymphadenopathy ,
arthritis ขอใดลดลง
1. Serum complement
2. Absolute white blood cell
3. Serum IgG 4. Lymphocyte
5. ESR
203. จากการศึกษาการเกิด leptospirosis ในไทยป 2548
อัตราอุบัติการณ = 0.4 และ อัตราการตาย = 0.02 ตอ
ประชาการแสนคน case fatality rate มีคาเทาไหร
a. 0.0002 b. 0.004
c. 0.4 d. 0.02 ตอประชากรแสนคน
e. 0.4 ตอประชากรแสนคน
204. ชาย 25 ป มีอาการปวดแสบใตลิ้นป ตอมาไปดื่ม
alcoholอยางมากจึงมาโรงพยาบาลผลการทําGastroscope
พบ Gastric mucosa บวม แดง Gastric biopsy พบ
Neutrophil จงบอกกลไกที่ทําใหเกิดพยาธิสภาพ
1. Autoimmune
2. Direct mucosal toxicity
3. Stimulate acid secretion
4. Alcohol-induced Thiamine deficiency
5. Stimulate growth of H.pylori
205. เกิด Point mutation ที Ribosome Subunit 50 ของ
Bacteria จะทําใหดื้อยาอะไรมากที่สุด
1. Cloxacillin 2. Ceftriaxone
3. Norfloxacin 4. Tetracycline
5. Erythromycin
206. ปวดชาตามปลายนิ้ว เมื่อโดนอากาศเย็น อากาศอุน
แลวดีขึ้น ถามวา involve เสนเลือดใด
1. Large artery
2. Medium artery
3. Small artery
4. Capillary
5. Venule
207. ชายอายุ 50 ป เปนเบาหวานมา 15 ป มาร.พ. ดวย
อาการขาบวมทั้งสองขางมา 2 เดือน ตรวจรางกาย BP
150/80 ตรวจปสสาวะ glucose 2+ protein 4+ RBC 2-3
cells/HPF WBC 1-2 cells/HPF granular cast 1-2/HPF
ไตนาจะมีสภาพอยางไร
1. nodular glomerulosclerosis
2. minimal-change glomerulonephritis
3. membranous glomerulonephrtis
4. membrano-proliferative glomerulonephritis
5. focal sclering glomerulonephritis
208. หญิง 25 ป อาชีพทําสวน กินยากําจัดวัชพืช หลังจาก
ทะเลาะกับสามี ตอมามีภาวะไตวายและการหายใจ
ลมเหลว เกิดจากการไดรับสารพิษใด
1. paraquat
2. organophosphate
3. ethylene glycol
4. arsenical compound
5. sodium hypochlorite
Basic Medical Science, March 2007
สหพันธสิตนักศึกษาแพทยแหงประเทศไทย
National Council of Thai Medical Students (NCTMS)
www.nctms.in.th
-26-
209. นักศึกษาแพทยชั้นปที่ 4 ไดรับการตรวจ tuberculin
test พบใหผลบวก ภาพรังสีทรวงอกปกติ การที่ภาพรังสี
ทรวงอกปกติ เปนผลมาจากระบบภูมิคุมกันใด
1. antibiotic 2. helper T cell
3. B lymphocyte 4. Innate immunity
5. Complement activation
210. เด็กชายอายุ 5 ป เปน nephrotic syndrome ไดรับ
prednisolone อยางตอเนื่องเปนเวลา 1 ป ควรหลีกเลี่ยง
การให vaccine ใดในผูปวยรายนี้
1. tetanus toxoid
2. varizella vaccine
3. influenza vaccine
4. hepatitis B vaccine
5. Japanese encephalitis vaccine
211.ชายอายุ 30 ป อาเจียนเปนเลือด ไดรับเลือด 30 cc มี
อาการไข หนาวสั่น แนนหนาอก BP80/60 อาการดัง
กลาวคืออะไร
1. anaphylaxis
2. citrate intoxication
3. bacterial endotoxic
4. immune complex reaction
5. antibody dependent cytotoxicity
212. ชายอายุ 60 ป มีอาการมือสั่นมา 2 เดือนเปนมากขึ้น
ขณะพัก ลําตัว rigid แขนไมแกวง ยาที่ใชรักษาโรคนี้ตองมี
ฤทธิ์อยางไรจึงจะดีที่สุดสําหรับโรคนี้
DA Cholinergic
a. เพิ่ม เพิ่ม
b. ลด ลด
c. เพิ่ม ลด
d. ลด เพิ่ม
213. ชายอายุ 25 ปจมน้ํา 5 นาที ไมรูสึกตัว ไมหายใจ
หลังทํา PCR หัวใจเตน และหายใจได แตก็ยังใชเครื่องชวย
หายใจอีก 20 ชั่วโมง ตอมารูสึกตัว ถามวา higher
cortical function ใดเสีย
1. นับเลข 2. บวกเลข
3. อานหนังสือ 4. จําชื่อญาติ
5. จําชื่อโรงพยาบาล
214.หญิงอายุ 30 ป เปน SLE ไดรับการรักษาดวยยา
corticosteroid มานาน เกิดภาวะแทรกซอน bacteria
infection หลายครั้ง ภาวะแทรกซอนที่เกิดขึ้นเกิดจาก
ความผิดปกติของกระบวนการใดในการกําจัดเชื้อกอโรค
a. ingestion b. attachment
c. chemotaxis d. degranulation
e. lysosome production
215. ผูปวยหญิงมีอาการปสสาวะแสบขัด ตรวจปสสาวะ
พบ E.coli ตองพบเชื้อจํานวนเทาใดในหนวย Colony
forming unit / mLจึงจะถือวาเปนเชื้อกอโรค
216. หญิงอายุ 17 ป นั่งรถไฟไปเชียงใหมมีอาการฉี่เปน
เลือด ผล LAB ........., Nitrite + หญิงคนนี้จะติดเชื้อใด
1. E.coli 2. Enterococcus Spp.
3. Trecomonas vaginalis
4. C.albicans
217. ผูปวยอายุ60ป มีปญหากินขาวแลวจุกหนาอก
เปนมา3เดือน ตอนแรกกลืนขาวแลวจุก ตอนหลังกลืน
ของเหลวไดอยางเดียว น้ําหนักลด 10กิโล สองกลองทาง
ปากพบกอนที่หลอดอาหารสวนกลาง ถาตัดมาดูจะพบวา
เปน
a. adenoma b. adenocarcinoma
c. squamous cell carcinoma
d. hemangioma e. cystadenocarcinoma
Basic Medical Science, March 2007
สหพันธสิตนักศึกษาแพทยแหงประเทศไทย
National Council of Thai Medical Students (NCTMS)
www.nctms.in.th
-27-
218. Organophosphate มีกลไกเกิดพิษอยางไร
1) ลดการสราง Ach
2) ลดการหลั่ง Ach
3) ลดการทํางานของ Ach esterase
4) ลดการ reuptake Ach
5) ลด sensitivity Ach receptor
219. เด็กชายอายุ 1 ป เปนโรคทางพันธุกรรมชนิดที่มีภาวะ
พรอง hydrolytic enzyme ถามวา enzyme ดังกลาวอยูใน
organelle ใด
A. acrosome
B. lysosome
C. ribosome
D. microsome
E. peroxysome
220. หญิงอายุ 45 ป ซีดและออนแรง ตรวจรางกายพบ
Cardiac murmur ที่ pulmonaric valve และมีคา Hct
20% อยากทราบวา Cardiac murmur เกิดจากอะไร
ก. หัวใจบีบตัวแรงขึ้น
ข. ปริมาตรเลือดเพิ่มขึ้น
ค. ความดันโลหิตสูงขึ้น
ง. ความเร็วการไหลของเลือดเพิ่มขึ้น
จ. ความตานทานหลอดเลือดเพิ่มขึ้น
221. หญิงอายุ35 ป มีกอนที่ทองดานขวาเมื่อผาตัดดูพบ
กอนที่รังไขดานขวาเมื่อผากอนออกดูพบผมและฟนกอนนี้
คืออะไร
ก) teratoma
ข) cisrotoma
ค) harmatoma
ง) chondoma
จ) cystadenoma
222. ผูหญิงอายุ50ป ทวมเล็กนอย เชื่องชา มีเสียงแหบ ...
ตรวจพบ TSH ต่ํา ทําTRH stimulation test พบวาTSH
เพิ่มขึ้นอยางชาๆ ถามวา อวัยวะใดผิดปกติ
a. Thyroid gl. b. Thalamus
c. Hypothalamus d. Anterior pituitary gl.
e. Posterior pituitary gl.
223. เด็กชายอายุ 5 ปไดรับการวินิจฉัยเปน glioma ขนาด
4 cm ที่ right frontal lobe ไดรับการรักษาโดย224. การ
ฉายรังสีจนครบพบวากอนยุบลงเล็กนอย cell มะเร็งมี
กลไกใดทําใหดื้อตอการรักษา
a. inactivation ของ p53 gene
b. inactivation ของ bcl-2 gene
c. inactivation ของ promoter of MDR-1 gene
d. activation ของ BCL-xl gene
e. activation ของ retinoblastoma gene
225. Blood pressure ลดลงจาก 110/70 เปน 100/70
เมื่อลุกขึ้นยืนเกิดจากสาเหตุใด
1. venous pooling ลดลง
2. stroke volumeลดงลง
ใชยาใดรักษา
1. praziquentel
2. albendazol
3. metronodazol
4. ivermectin
5. amphotericin B
2007821172158 466 6438_1
2007821172158 466 6438_1
2007821172158 466 6438_1

More Related Content

What's hot

Compre si 2010 ans
Compre si 2010 ansCompre si 2010 ans
Compre si 2010 ans
vora kun
 
Nt2009 complete all
Nt2009 complete allNt2009 complete all
Nt2009 complete all
vora kun
 
Mdcu Preventive Screening
Mdcu Preventive ScreeningMdcu Preventive Screening
Mdcu Preventive Screening
vora kun
 
Osce ศรว ครั้งแรก dec52
Osce ศรว ครั้งแรก dec52Osce ศรว ครั้งแรก dec52
Osce ศรว ครั้งแรก dec52
vora kun
 
Step3 Tutorial by SWU book1
Step3 Tutorial by SWU book1Step3 Tutorial by SWU book1
Step3 Tutorial by SWU book1
vora kun
 
Nt2009 Complete Ans
Nt2009 Complete AnsNt2009 Complete Ans
Nt2009 Complete Ans
vora kun
 
การซักประวัติการเจ็บป่วย
การซักประวัติการเจ็บป่วยการซักประวัติการเจ็บป่วย
การซักประวัติการเจ็บป่วย
Ozone Thanasak
 

What's hot (17)

Compre si 2010 ans
Compre si 2010 ansCompre si 2010 ans
Compre si 2010 ans
 
Nt2009 complete all
Nt2009 complete allNt2009 complete all
Nt2009 complete all
 
Part 2 - PD guideline Dialysis Weekend 9-11 feb 2018_pattaya
Part 2 - PD guideline  Dialysis Weekend 9-11 feb 2018_pattayaPart 2 - PD guideline  Dialysis Weekend 9-11 feb 2018_pattaya
Part 2 - PD guideline Dialysis Weekend 9-11 feb 2018_pattaya
 
Nl part ii march 2009
Nl part ii march 2009Nl part ii march 2009
Nl part ii march 2009
 
For extern
For externFor extern
For extern
 
กรณีศึกษาไต (Ns) แก้ไข
กรณีศึกษาไต (Ns)  แก้ไขกรณีศึกษาไต (Ns)  แก้ไข
กรณีศึกษาไต (Ns) แก้ไข
 
Mdcu Preventive Screening
Mdcu Preventive ScreeningMdcu Preventive Screening
Mdcu Preventive Screening
 
แนวปฏิบัติ การล้างไตทางช่องท้อง พ.ศ. 2561
แนวปฏิบัติ การล้างไตทางช่องท้อง พ.ศ. 2561แนวปฏิบัติ การล้างไตทางช่องท้อง พ.ศ. 2561
แนวปฏิบัติ การล้างไตทางช่องท้อง พ.ศ. 2561
 
Osce ศรว ครั้งแรก dec52
Osce ศรว ครั้งแรก dec52Osce ศรว ครั้งแรก dec52
Osce ศรว ครั้งแรก dec52
 
Case study surgery
Case study surgeryCase study surgery
Case study surgery
 
Step3 Tutorial by SWU book1
Step3 Tutorial by SWU book1Step3 Tutorial by SWU book1
Step3 Tutorial by SWU book1
 
Ebola virus disease
Ebola virus disease Ebola virus disease
Ebola virus disease
 
Ebola virus disease
Ebola virus disease Ebola virus disease
Ebola virus disease
 
Nt2009 Complete Ans
Nt2009 Complete AnsNt2009 Complete Ans
Nt2009 Complete Ans
 
หนังสือความรู้เรื่องโรคไต
หนังสือความรู้เรื่องโรคไตหนังสือความรู้เรื่องโรคไต
หนังสือความรู้เรื่องโรคไต
 
Example osce
Example osceExample osce
Example osce
 
การซักประวัติการเจ็บป่วย
การซักประวัติการเจ็บป่วยการซักประวัติการเจ็บป่วย
การซักประวัติการเจ็บป่วย
 

Viewers also liked

ตาราง Intro 2
ตาราง Intro 2ตาราง Intro 2
ตาราง Intro 2
New Srsn
 
โรคถุงลมโป่งพอง
โรคถุงลมโป่งพองโรคถุงลมโป่งพอง
โรคถุงลมโป่งพอง
Wan Ngamwongwan
 
Acute glomerulonephritis
Acute glomerulonephritisAcute glomerulonephritis
Acute glomerulonephritis
New Srsn
 
COPD.ppt
COPD.pptCOPD.ppt
COPD.ppt
Shama
 
Mechanical Ventilation in ARDS vs COPD
Mechanical Ventilation in ARDS vs COPDMechanical Ventilation in ARDS vs COPD
Mechanical Ventilation in ARDS vs COPD
cairo1957
 
Chronic obstructive pulmonary disease
Chronic obstructive pulmonary diseaseChronic obstructive pulmonary disease
Chronic obstructive pulmonary disease
imangalal
 

Viewers also liked (18)

Nl 2010 nctms
Nl 2010 nctmsNl 2010 nctms
Nl 2010 nctms
 
ตาราง Intro 2
ตาราง Intro 2ตาราง Intro 2
ตาราง Intro 2
 
TAEM10:Respiratory distress
TAEM10:Respiratory distressTAEM10:Respiratory distress
TAEM10:Respiratory distress
 
สปสช.การบริหารAsthma&copd57
สปสช.การบริหารAsthma&copd57สปสช.การบริหารAsthma&copd57
สปสช.การบริหารAsthma&copd57
 
งานนำเสนอ2
งานนำเสนอ2งานนำเสนอ2
งานนำเสนอ2
 
Biochemistry tutorial
Biochemistry tutorialBiochemistry tutorial
Biochemistry tutorial
 
โรคถุงลมโป่งพอง
โรคถุงลมโป่งพองโรคถุงลมโป่งพอง
โรคถุงลมโป่งพอง
 
ศรว 51 ANS By Cmu
ศรว 51 ANS By Cmuศรว 51 ANS By Cmu
ศรว 51 ANS By Cmu
 
รวมข้อสอบCompre nl
รวมข้อสอบCompre nlรวมข้อสอบCompre nl
รวมข้อสอบCompre nl
 
Acute glomerulonephritis
Acute glomerulonephritisAcute glomerulonephritis
Acute glomerulonephritis
 
3 Asthma and COPD Management
3 Asthma and COPD Management3 Asthma and COPD Management
3 Asthma and COPD Management
 
COPD GOLD 2014
COPD GOLD 2014COPD GOLD 2014
COPD GOLD 2014
 
COPD.ppt
COPD.pptCOPD.ppt
COPD.ppt
 
Mechanical Ventilation in ARDS vs COPD
Mechanical Ventilation in ARDS vs COPDMechanical Ventilation in ARDS vs COPD
Mechanical Ventilation in ARDS vs COPD
 
Chronic obstructive pulmonary disease
Chronic obstructive pulmonary diseaseChronic obstructive pulmonary disease
Chronic obstructive pulmonary disease
 
Copd update 2015
Copd update 2015Copd update 2015
Copd update 2015
 
COPD (Chronic Obstructive Pulmonary Disease)
COPD (Chronic Obstructive Pulmonary Disease)COPD (Chronic Obstructive Pulmonary Disease)
COPD (Chronic Obstructive Pulmonary Disease)
 
COPD (Chronic obstructive Pulmonary Disease) PowerPoint Presentation -aslam
COPD  (Chronic obstructive Pulmonary Disease) PowerPoint Presentation -aslamCOPD  (Chronic obstructive Pulmonary Disease) PowerPoint Presentation -aslam
COPD (Chronic obstructive Pulmonary Disease) PowerPoint Presentation -aslam
 

Similar to 2007821172158 466 6438_1

Exercise national license_part_ii_march_2009_2
Exercise national license_part_ii_march_2009_2Exercise national license_part_ii_march_2009_2
Exercise national license_part_ii_march_2009_2
Loveis1able Khumpuangdee
 
Osce examination si116
Osce examination si116Osce examination si116
Osce examination si116
vora kun
 
กาย่า presentation
กาย่า presentationกาย่า presentation
กาย่า presentation
Crystalpet
 
Nle step 2_2009 si115-116 and nle_step_2_2009 nctms editors cut
Nle step 2_2009 si115-116 and nle_step_2_2009 nctms editors cutNle step 2_2009 si115-116 and nle_step_2_2009 nctms editors cut
Nle step 2_2009 si115-116 and nle_step_2_2009 nctms editors cut
Loveis1able Khumpuangdee
 
Case esrd วรงค์วุฒิ 11 ส.ค. 53
Case esrd  วรงค์วุฒิ  11 ส.ค. 53Case esrd  วรงค์วุฒิ  11 ส.ค. 53
Case esrd วรงค์วุฒิ 11 ส.ค. 53
Watcharapong Rintara
 
Water_for_body.pdf
Water_for_body.pdfWater_for_body.pdf
Water_for_body.pdf
60919
 
Pat2 52 72
Pat2 52 72Pat2 52 72
Pat2 52 72
june41
 
ข้อสอบ Pat 2 (ฟิสิกส์ เคมี ชีวะ)
ข้อสอบ Pat 2 (ฟิสิกส์ เคมี ชีวะ)ข้อสอบ Pat 2 (ฟิสิกส์ เคมี ชีวะ)
ข้อสอบ Pat 2 (ฟิสิกส์ เคมี ชีวะ)
Lupin F'n
 

Similar to 2007821172158 466 6438_1 (20)

Rr rx
Rr rxRr rx
Rr rx
 
Nl part ii march 2009
Nl part ii march 2009Nl part ii march 2009
Nl part ii march 2009
 
Exercise national license_part_ii_march_2009_2
Exercise national license_part_ii_march_2009_2Exercise national license_part_ii_march_2009_2
Exercise national license_part_ii_march_2009_2
 
Osce examination si116
Osce examination si116Osce examination si116
Osce examination si116
 
Centre management - PD quality นพ.สกานต์
Centre management -  PD quality นพ.สกานต์Centre management -  PD quality นพ.สกานต์
Centre management - PD quality นพ.สกานต์
 
National license 2010 by med tu 16
National license 2010 by med tu 16National license 2010 by med tu 16
National license 2010 by med tu 16
 
คู่มือการดูแลผู้ป่วยล้างไตทางช่องท้องสำหรับประชาชน 2557
คู่มือการดูแลผู้ป่วยล้างไตทางช่องท้องสำหรับประชาชน 2557คู่มือการดูแลผู้ป่วยล้างไตทางช่องท้องสำหรับประชาชน 2557
คู่มือการดูแลผู้ป่วยล้างไตทางช่องท้องสำหรับประชาชน 2557
 
Hand out km & best practice อุดรธานี พฤศจิกายน 2557
Hand out km & best practice อุดรธานี พฤศจิกายน 2557Hand out km & best practice อุดรธานี พฤศจิกายน 2557
Hand out km & best practice อุดรธานี พฤศจิกายน 2557
 
การป้องกันโรคไตแบบบูรณาการ “งานของหมอและประชาชน”
การป้องกันโรคไตแบบบูรณาการ “งานของหมอและประชาชน”การป้องกันโรคไตแบบบูรณาการ “งานของหมอและประชาชน”
การป้องกันโรคไตแบบบูรณาการ “งานของหมอและประชาชน”
 
กาย่า presentation
กาย่า presentationกาย่า presentation
กาย่า presentation
 
Nle step 2_2009 si115-116 and nle_step_2_2009 nctms editors cut
Nle step 2_2009 si115-116 and nle_step_2_2009 nctms editors cutNle step 2_2009 si115-116 and nle_step_2_2009 nctms editors cut
Nle step 2_2009 si115-116 and nle_step_2_2009 nctms editors cut
 
Case esrd วรงค์วุฒิ 11 ส.ค. 53
Case esrd  วรงค์วุฒิ  11 ส.ค. 53Case esrd  วรงค์วุฒิ  11 ส.ค. 53
Case esrd วรงค์วุฒิ 11 ส.ค. 53
 
โปรแกรมตรวจสุขภาพ 13 รายการ 999 บาท
โปรแกรมตรวจสุขภาพ 13 รายการ 999 บาทโปรแกรมตรวจสุขภาพ 13 รายการ 999 บาท
โปรแกรมตรวจสุขภาพ 13 รายการ 999 บาท
 
Epilepsy
EpilepsyEpilepsy
Epilepsy
 
Present blood program
Present blood programPresent blood program
Present blood program
 
Water_for_body.pdf
Water_for_body.pdfWater_for_body.pdf
Water_for_body.pdf
 
CKD for 2019
CKD for 2019 CKD for 2019
CKD for 2019
 
Pat2 52 72
Pat2 52 72Pat2 52 72
Pat2 52 72
 
Pat2 2552
Pat2 2552Pat2 2552
Pat2 2552
 
ข้อสอบ Pat 2 (ฟิสิกส์ เคมี ชีวะ)
ข้อสอบ Pat 2 (ฟิสิกส์ เคมี ชีวะ)ข้อสอบ Pat 2 (ฟิสิกส์ เคมี ชีวะ)
ข้อสอบ Pat 2 (ฟิสิกส์ เคมี ชีวะ)
 

2007821172158 466 6438_1

  • 1. Basic Medical Science, March 2007 สหพันธสิตนักศึกษาแพทยแหงประเทศไทย National Council of Thai Medical Students (NCTMS) www.nctms.in.th -1- แนวขอสอบ Basic Medical Science March, 2007 สหพันธนิสิตนักศึกษาแพทยแหงประเทศไทย ประจําปการศึกษา 2550 รุนที่ 17 1. ผูหญิงคนหนึ่งไปเสริมนม silicone ทําแลวนมผิดรูป อาการนี้เกิดจากเซลลใดเกี่ยวของ ก. T lymphocyte ข. B lymphocyte ค. Histiocyte ง. PMN จ. Basophil 2. ชายอายุ 50 ป ดื่มสุราเปนประจํามาเปนเวลา 10 ป มี อาการปวดทองบริเวณใตลิ้นป บางครั้งปวดราวทะลุไป ดานหลัง ถายอุจจาระเปนมูกมัน ไมมีตาเหลือง ตาเหลือง อาการดังกลาวสัมพันธกับพยาธิสภาพที่อวัยวะใด ก. liver ข. stomach ค. pancrease ง. duodenum จ. gall bladder 3. เด็กชายอายุ 10 ป กลืนลําบากเนื่องจาก lower esophagus sphinter ทํางานผิดปกติ ถาทํา manometry LES pressure จะเปนอยางไร ก. สูงกวาปกติในพัก ข. สูงกวาปกติในขณะกลืน ค. ต่ํากวาปกติในขณะผัก ง. ต่ํากวาปกติในขณะกลืน จ. ต่ํากวาปกติในกลืน และสูงกวาปกติในขณะพัก 4. เด็กชายอายุ 3 ป ผิวแหง ผิวลอก เปนโรคขาดสารอาหาร แบบ kwarsiokor อยากทราบวาจะมี amino acid ตัวไหน ปกติ a. Threonine b. Tryptophan c. Methionine d. Aspartic acid e. Phenyl alanine 5. การทํา mediolateral episiotomy หลีกเลี่ยงอันตราย ตอ muscle ใด ก. bulbospongiosus ข. ischicavernosus ค. superficial peroneal nerve ง. deep peroneal nerve จ. external anal sphinter 6. ผูปวยหญิงอายุ 17 ป มาพบแพทยเนื่องจากไมมี ประจําเดือน มีขนาดหนาอกปกติ pubic hair ปกติ ตรวจ ชองคลอดพบวามี blind vaginal pouch และมีกอน เคลื่อนที่ไดบริเวณ labia majora ตรวจ ultrasound ไมพบ uterus ผูปวยนาจะเปนโรคใด a. Turner syndrome b. Gonadal dysgenesis c. Adenogenital syndrome d. Female pseudohermaphrodite e. Testicular feminizing syndrome
  • 2. Basic Medical Science, March 2007 สหพันธสิตนักศึกษาแพทยแหงประเทศไทย National Council of Thai Medical Students (NCTMS) www.nctms.in.th -2- 7. ผูปวยเด็ก 5 ป เปน glioma ขนาด 4 cm ที่ Rt. Frontal lobe ไดรับการฉายรังสี กอนเล็กลงเล็กนอย อะไรทําใหดื้อ ตอการรักษา ก. inactivation of bcl2 gene ข. inactivation of p53 gene ค. inactivation of precursor of MDL-2 gene ง. activation of BCL-XL gene จ. activation of retinoblastoma gene 8. คนไขชายมีอาการหอบ ตามัว จากการดื่มสุราเถื่อน หมอให ethanol แกผูปวยเพื่อรักษา กลไกการออกฤทธิ์ ของ ethanol คือ ก. Product inhibitor ข. Allosteric inhibitor ค. Irreversible inhibitor ง. Competitive inhibitor จ. Feedback inhibitor 9. กลไกของยา cyclosporine จะยับยั้งการทํางานของ cell ใด ก. B-cell ข. T-cell ค. Macrophage ง. NK cell จ. PMN 10. เด็กชายอายุ 2 ป เดินชนสิ่งของบอยๆ ตอนกลางคืน ตรวจรางกาย พบ Biot’s spot (เกล็ดกระดี่) ที่ตา อาการ ของผูปวยเกิดจากการขาดสารในขอใด ก. Retinol ข. Thiamine ค. Riboflavin ง. Pyridoxine จ. Ascorbic acid 11. Urinalysis ตรวจตอนไหนเปนอยางชาจะไดผล lab ที่ เชื่อถือได ถาเก็บตอน 14.00 น. ก. 14.30 ข. 15.00 ค. 16.00 ง. 18.00 จ. 20.00 12.หญิงอายุ 60 ป อาเจียนอยางรุนแรงและถายอุจจาระ เปนน้ํา 5 ครั้ง ใน 6 ชั่วโมง ตรวจรางกายพบ postural hypotension ริมฝปากแหง poor skin turgor รางกายของ ผูปวยจะมีการเปลี่ยนแปลงอยางไร a. การหลั่ง aldosterone ลดลง b. ระดับ plasma angiotensin II ต่ํา c. การหลั่ง ANP เพิ่ม d. Sympathetic activity เพิ่มขึ้น e. Efferent arteriole resistant ที่ไตเพิ่มขึ้น 13. เด็กหญิงอายุ 2 ป ถายอุจจาระเปนน้ําวันละ 10 ครั้ง มา 2 วัน ตรวจ serum CO2 = 12 mmol/L urine pH= 5.0 การเปลี่ยนแปลงทางหองปฏิบัติการดังกลาว เกิดจากการ compensation ที่ตําแหนงใดของ nephron a. Glomerulus b. Proximal tubule c. Loop of henle d. Distal tubule e. Collecting duct 14. พิษสุราเรื้อรัง ชาปลายมือปลายเทา deep tendon reflex แขนขาลดลง metabolic pathway ใดผิดปกติ a. glycolysis b. Kreb’s cycle c. Glucogenesis d. Glycogenesis e. Oxidative
  • 3. Basic Medical Science, March 2007 สหพันธสิตนักศึกษาแพทยแหงประเทศไทย National Council of Thai Medical Students (NCTMS) www.nctms.in.th -3- 15. ผูปวยเด็กเปน acute lymphoblastic leukemia ไดรับ ยาเคมีบําบัด มีภาวะ uric acid สูง metabolism สารใด ผิดปกติ a. heme b. purine c. pyrimidine d. amino acid e. glucoronide 16. ผูปวยชายอายุ 55 ป ถายปสสาวะเปนสีเลือด จากการ ตรวจชิ้นเนื้อ พบวาเปน transitional cell carcinoma ที่ กระเพาะปสสาวะ ผูปวยคนนี้นาจะมีพฤติกรรมตรงกับขอ ใด a. สูบบุหรี่จัด b. อั้นปสสาวะบอย c. ประกอบอาชีพเกษตรกรรม d. ดื่มสุรามาก e. ทานอาหารปงยาง 17. ชายอายุ 50 ป ดื่มเหลาเปนประจํา 10 ป ปวดบริเวณ epigastric ราวไปที่หลัง ถายเปนมูกมัน ตับมามไมโต มี ความผิดปกติที่อวัยวะไหน a. stomach b. liver c. pancrease d. small intestine e. gallbladder 18. ชายอายุ 15 ป เตะฟุตบอล หายใจเร็วและแรงขึ้น กลไกใดทําให ไดรับอากาศใหมมากที่สุด a. Tidal volume b. Respiratory rate c. Minute ventilation d. Dead space ventilation e. Inspire reserve volume 19. ชายอายุ 50 ป วินิจฉัยเปน pleural effusion ตองเจาะ effusion มาตรวจ โดยเจาะระหวาง ant. Axillary line กับ pleural reflexion ตําแหนงที่เจาะอยูระดับ rib ใด a. 7 b. 8 c. 9 d. 10 e. 11 20. ชายอายุ 35 ป หลังอดน้ําและอาหารเปนเวลา 16 ชม. เพื่อเตรียมรับการผาตัด มีระดับ urine sp.gr 1.025 อยากทราบวาระดับ hormone ใดในเลือดเพิ่มขึ้น a. insulin b. Oxytocin c. Vasopressin d. Glucocorticoid e. ANP 21. หญิงไทยมีอาการออนเพลีย ปวดขอ เปนแผลที่ปาก เปนๆหายๆ มาเปนเวลานาน ตรวจพบ antinuclear ab titer 1:512 ควรตรวจอะไรเพิ่มเพื่อ diagnosis โรค a. anti ds DNA ab. b. SER c. CD4 T-cell d. CBC e. ANA 22. อัตราการตายของผูใหญอายุ 70 ป เปน 0.1/ป อัตรา การตายของผูปวยโรคหัวใจเปน 0.08/ป อัตราการตายของ ผูปวยโรคมะเร็งลําไสเปน 0.02/ป ถาม life expectation ของผูใหญอายุ 70 ป ที่เปนโรคหัวใจและมะเร็งลําไส a. 1 b. 2 c. 5 d. 8 e. 10
  • 4. Basic Medical Science, March 2007 สหพันธสิตนักศึกษาแพทยแหงประเทศไทย National Council of Thai Medical Students (NCTMS) www.nctms.in.th -4- 23. ความสัมพันธระหวาง FEV1 (force expiration in 1 second) กับอายุ เปนดังสมการ FEV1(liter) = (1.052 x height) - (0.244 x age) -0.561 ขอใดกลาวถูกตอง a. ถาอายุขัยลดลง 1 ป ,FEV1 จะลดลง 0.244 liter b. ถาอายุขัยลดลง 0.244 ป FEV1 จะลดลง 1 liter c. ถาอายุเพิ่มขึ้น 1ป FEV1 จะลดลง 0.244 liter d. ถาอายุเพิ่มขึ้น 0.244 ป FEV1 จะลดลง 1 liter 24. เด็กชายอายุ 5 ป มีความผิดปกติของ LES (lower esophageal sphincter) ถาทํา esophageal manometry จะพบภาวะใด ก. ต่ําลงขณะพัก ข. ต่ําลงขณะกลืน ค. เพิ่มขณะพัก ง. เพิ่มขณะกลืน จ. เพิ่มขณะพัก&ลดขณะกลืน 25. ชาย 25 ป ฝกทหารกลางแดดเปนเวลา 2 ชั่วโมง ติดตอกัน มีอาการหนาแดง ตัวรอน ตัวแหง ไมมีเหงื่อและ หมดสติ ควรทําอยางไรเปนการรักษาอาการไดดีที่สุด a. ใชพัดลมเปา b. ใหนอนยกเทา c. ใชน้ําเย็นราดใหโชกทั้งตัว d. กันไมใหคนมุง e. ใชผาหอน้ําแข็งวางบนหนาผาก 26. ชายอายุ 50 ป เสียชีวิตในรถยนตที่ติดเครื่องและเปด เครื่องปรับอากาศ สภาพศพผิวหนัง มีสีแดงสด ญาติเลา ประวัติวา ผูตายนอนในรถนานกวา 5 ชม. สภาพผิวหนังสี แดงสด เกิดจาก Hb อะไร a. sulfhemoglobin b. methamoglobin c. oxyhemoglobin d. carboxyhemoglobin e. oxidized hemoglobin 27. สูบบุหรี่มานาน หอบเหนื่อยงายขอใดถูก a. FEV1/FRC ลด b. Elascity ลด c. Compliance ลด d. Restrictive fibers 28. ชายอายุ 20 ป ปวดที่ขอสะโพก แพทยพบวาเปน posterior dislocation จงบอกวามีการเคลื่อนไหวอยางใด ที่ hip joint จึงทําใหเกิดอาการขึ้น a. flex, medial rotate, adduct b. flex, medial rotate, adduct c. flex, lateral rotate, abduct d. extend, lateral rotate, abduct e. extend, medial rotate, abduct 29. ชายอายุ 25 ป มีโพรงหนองที่แขนและนอง แพทย วินิจฉัยวาเปน polymyositis เจาะหนองออกมายอมเชื้อ พบวาเปน bacterial ทรงกลม ทดสอบ coagulase และ catalase ใหผลเปนบวก นาจะเปนเชื้อใด a. Stap. Aureus b. Stap. Epidermidis c. Stap. Haemolyticus d. Strep pyogenes e. Strep. Pneumoniae 30. กินกลวยบวดชีกอนบริจาคเลือด พบวาเลือดขุนขน จากอะไร a. micelle b. chylomicron c. cholesterol d. fatty acid e. monoglycerine
  • 5. Basic Medical Science, March 2007 สหพันธสิตนักศึกษาแพทยแหงประเทศไทย National Council of Thai Medical Students (NCTMS) www.nctms.in.th -5- 31. แพทยทําคลอด สังเกตวามีน้ําคร่ํามากกวาปกติ หลังจากนั้น 1 วัน มีอาการสําลักนม และไมสามารถกลืน อาหารได ไมสามารถใสทออาหาร( NG tube ) มีความ พิการแตกําเนิดอะไร a. midgut volvulus b. duodenal atresia c. annular pancrease d. hypertrophic pyrolic stenosis e. esophageal atresia 32. ผูชายอายุ 30 ป tidal volume 500 mL,anatomical dead space 200 mL, RR15 ถาเปลี่ยน RR เปน30,tidal volume 350 แต anatomical dead space เทาเดิมจะมี ผลเปลี่ยนแปลงอยางไร a. Paco2 b. Paco2 c. paco2ไมเปลี่ยนแปลง d. pH e. pHลด 33. ชายอายุ 70 ป สูบบุหรี่มา 40 ป มีอาการไอเรื้อรัง บางครั้งหอบมา2ป x-ray ปรากฏวาเปน emphysema คา ใดจะลดลง a. airway resistance b. lung compliance c. intrapleural P d. FRV e. FEV1 34. ถารางกายขับ bile salt มากขึ้นถามวา สารในขอใด ในรางกายลดลง a. bilirubin b. lecithin c. fatty acid d. triglyceride e. cholesterol 35. ผูปวยชายอายุ 50ป สูบบุหรี่จัด มาที่หองฉุกเฉินดวย อาการเจ็บหนาอกมา 1 ชั่วโมง สง lab พบวา creatinine kinase และ troponin-t เพิ่มขึ้น ถามวาเกิดอะไรขึ้น a. dissociation of ribosome b. swelling ของmitochodria c. clumpingของ nuclear chromatin d. Fragmentation ของ plasma membrane 36. ชายอายุ 30ป ไมมีโรคประจําตัว ปวดศีรษะอยางรุนแรง ขณะทํางาน ไมเคยปวดแบบนี้มากอน ไมมีอาการนํามากอน ตรวจรางกายพบ stiff neck และ papilledema ไมมีอาการ muscle weakness ลักษณะแสดงถึงพยาธิสภาพใด a. brain abscess b. hydrocephalus c. Thrombotic stroke d. acute bacterial meningitis e. Ruptured cerebral aneurism 37. หญิงอายุ60ป มีประวัติเปนความดันโลหิตสูงและ เบาหวานมา10ป หลังตื่นนอนมีอาการแขนขาขวาออนแรง และพูดไมได ตรวจรางกายพบright hemiparesis with motor aphasia มีการอุดตันที่หลอดเลือดใด a. anterior cerebral artery right b. anterior cerebral artery left c. middle cerebral artery right d. middle cerebral artery left e. posterior cerebral artery right 38. ผูปวยหญิงตรวจรางกาย ที่ปากมดลูกพบ red vesicle,erythema and papule เปนกลุมๆคลําพบตอม น้ําเหลืองโต 2 ขาง กดเจ็บ ตรวจ ทาง histopathology พบ intranuclear halo ,multinucleated giant cell เชื้อที่ เปนไปไดคือ a. HSV b. Gonococcal infection c. CMV d. Treponema pallidum e. HPV
  • 6. Basic Medical Science, March 2007 สหพันธสิตนักศึกษาแพทยแหงประเทศไทย National Council of Thai Medical Students (NCTMS) www.nctms.in.th -6- 39. นายวิชัย และนายวิวัฒนวิ่งแขงขันกีฬามหาวิทยาลัย โดยนายวิชัยแขงวิ่ง 200 m นายวิวัฒนแขงวิ่งมาราธอน ถามวาระดับปริมาณของสารใดในกลามเนื้อขาของนาย วิชัยที่มีมากกวานายวิวัฒน ก. จํานวน mitochondria ข. ระดับglycolytic enzyme ค. muscle blood flow ง. Fat composition 40. หญิง 3 5ป มีกอนที่หนาอกขวามา1m กอนโตขึ้น กด เจ็บ กอนมีขนาด 6 cm hard consistency มี bloody discharge ออกจาก nipple คลําที่ lymph node ดานขวา พบกอนขนาด 4cm ขอใดบงลักษณะของมะเร็ง ก. กอนขนาด 6 cm ข. กดเจ็บ ค. มี bloody discharge ออกมา ง. คลําพบกอนที่ LN จ. Hard consistency 40. ขอใดมีโอกาสเกิด resolution มากที่สุด ก. ruptured spleen ข. skin ulcer ค. lobar pneumonia ง. bowel gangrene จ. chronic viral hepatitis 41. ผูปวยหญิงอายุ 35 ป ตาโปน ใจสั่น เหนื่อยงาย diffuse enlargement of thyroid gland, free T4 สูงขึ้น การตรวจ lab ใดสําคัญที่สุดในการวินิจฉัยโรค ก. thyroid scan ข. triiodothyroxine ค. thyroxine binding gobulin ง. TSH จ. TRH 42. ชายอายุ 35 ป ไมเคยออกกําลังกาย ออกกําลังกาย ครั้งแรกเสียเหงื่อมาก กอนออกกําลังและหลังออกกําลังมี อาการปกติ ถามวา rennin-angiotensin system มีกลไก การรักษา BP อยางไร a. เพิ่ม GFR b. เพิ่ม renal blood flow c. เพิ่ม Na reabsorption d. ลด ACE activity 43. หญิง BP ตก มีประวัติทานยาหมอ ตรวจรางกายอื่นๆ มี buffalo hump หากเจาะดูจะพบ serum electrolyte ยังไง ก. hyponatremia ข. hypokalemia ค. hypocalcemia ง. metabolic alkalosis จ. hyperuricemia 44. ผูชายมาดวยอาการปวดทองนอยขวา ตรวจพบ acute appendicitis ซึ่งมีการ inflammation แบบใด a. serous inflammation b. organized inflammation c. fibropurulent inflammation d. granulomatous inflammation 45. มี abscess ที่ขาขวานํามาตรวจพบ Gram +ve cocci in cluster และมี Lymph node ที่ขาหนีบโต มี pathology แบบใด a. sinus histiocytosis b. follicular hyperplasia c. parafollicular hyperplasia d. granulomatous hyperplasia e. interfollicular hyperplasia
  • 7. Basic Medical Science, March 2007 สหพันธสิตนักศึกษาแพทยแหงประเทศไทย National Council of Thai Medical Students (NCTMS) www.nctms.in.th -7- 46. หญิง 30 ป เสริมหนาอกดวย silicone 1 ปตอมา คลํา ไดกอนแข็งที่เตานมขางขวาและเตานมผิดรูป เซลลใดที่มี บทบาทสําคัญที่ทําใหเกิดพยาธิสภาพนี้ a. Basophil b. Eosinophil c. Histiocyte d. Neutrophil e. Plasma cell 47. ชายอายุ 17 ป รางกายปกติ กินเนื้อเค็มและน้ําตาม a. ลดการกระตุน Baroreceptor b. ลดการกระตุน Low Pressure receptor c. เพิ่ม rennin d. ลดการกระตุน chemoreceptor e. เพิ่ม Hydrostatic pressure 48. เมื่อเรากินเค็ม อะไรเพิ่ม a. Na+ เพิ่ม b. K+ เพิ่ม c. Renin เพิ่ม d. Aldosterone เพิ่ม e. ANP เพิ่ม 49. ยา Nanosum ไดรับอนุญาตใหวางขาย ตอมาพบวา คนใชยาไดรับผลขางเคียง ขั้นตอนใดที่สามารถระงับการ วางจําหนายตอไปได a. Phase IIa Clinical trial b. Phase IIb Clinical trial c. Phase III Clinical trial d. Phase IV Clinical trial 50. เด็ก 2 ป ถายเปนน้ํา serum CO2 Urea pH 5.0 nephron มี compensate ที่ไหน ก. Glomerulus ข. Proximal tubule ค. Loop of Henle ง. Distal tubule จ. Collecting duct 51. หญิงหมาย 50 ป หมดประจําเดือนมี Fasting blood glucose 200 Triglyceride 200 HDL 35 LDL 150 จะ เสี่ยงตอการเปนโรคอะไร a. Alzheimer b. Depression c. Coronary heart disease d. Cerebrovascular disease 52. ชายอายุ 40 ป ไมมีโรคประจําตัว พบ Hct 60% WBC 6,000 platelet 200,000 สัมพันธกับขอใด ก. อวน ข. ดื่มสุรา ค. สูบบุหรี่ ง. ยาฆาแมลง จ. ทํางานในโรงงานอุตสาหกรรม 53. ชายอายุ 20 ป ถูกแทงที่หนาทอง แพทยไดทําการ ผาตัดหนาทองลงไปถึงชั้น transverse abdominis อยาก ทราบวาโครงสรางใดที่อยูถัดจากชั้นนี้ a. external abdominis muscle b. Internal abdominis muscle c. Rectus abdominis d. Transversalis fascia e. Peritoneum
  • 8. Basic Medical Science, March 2007 สหพันธสิตนักศึกษาแพทยแหงประเทศไทย National Council of Thai Medical Students (NCTMS) www.nctms.in.th -8- 54. ชายอายุ 20 ป ดําน้ําดูปะการัง กอนดําหายใจลึก 5-6 ครั้ง คอยๆขึ้นผิวน้ําชาแลวหมดสติ ถามวาเพราะอะไร ก. Hypoxia ข. Hyperoxia ค. Hypocapnia ง. Hypercapnia จ. Pneumothorax 55. ชาย 25 ป รถชน หมดสติ หัวมีบาดแผล ตรวจ รางกายได HR 66 RR 20 Pressure 210/100 ถามวา ปจจัยทาง CVS ใดที่มีผลตอความดันที่สุด ก. Blood volumn ข. Preload ของ หัวใจ ค. Afterload ของ หัวใจ ง. Sym ที่หัวใจ จ. Peripheral Vasoconstriction 56. ชาย 20 ป ไขต่ํา 8 สัปดาห ตรวจรางกายพบ murmur ตรวจ echo หัวใจ พบ 0.5 cm vegetative ที่ mitral เพาะเชื้อ 7 วัน ไมขึ้น Bacteria ถามวาเชื้อใด ก. E.faecalis ข. S.Aureus ค. H.aphrophilus ง. P.aeruginosa จ. S.epidermidis 57. บริเวณที่ใชเปนตําแหนงทํา Bone Marrow biopsy ใน ผูใหญ อยูที่ใด a. Tibial b. Humerus c. Radius d. Femur e. Iliac spine 58. ยาเกามีประสิทธิภาพในการรักษา 60 ยาใหมมี ประสิทธิภาพ 80 คา Number needed to treat เทาไหร ก. 5 ข. 20 ค. 33.3 ง. 50 จ. 10 59. ผูชายอายุ 25 ป มาดวยอาการไข ไอหายใจเหนื่อย หอบ มา 2 วัน แพทยตรวจเสมหะพบ Gram +ve diplococci แพทยใหการวินิจฉัยวาเปน Pneumonia Criteria ที่ประเมินวา sputum gram strain นี้นาเชื่อถือ Epithelium (/HPF) WBC(/HPF) a. 10 40 b. 25 30 c. 30 50 d. 50 10 e. 30 40 60. ชาย 50 ป เปนโรคตับแข็ง มี Ascites ไดรับยา furosemide 1 wk ตอมามีอาการออนแรง เวียนศีรษะ สับสน เปนตะคริว ตรวจเลือดผล Na 130 K 2.5 pH 7.58 PCO2 50 ความผิดปกติสมดุล acid-base เปนแบบใด a. uncompensated respiratory alkalosis b. compensated respiratory alkalosis c. uncompensated metabolic acidosis d. compensated metabolic acidosis e. mixed respiratory and metabolic alkalosis 61. ชาย 25 ป ไข ไอหอบ มา 2 เดือน หมอวินิจฉัยวาเปน pneumonia ยอม gram พบ positive diplococci ดูความ นาเชื่อถือ ของเชื้อใน sputum วาอยางไร adequate 62. Terbutaline sulfate ออกฤทธิ์ยังไง a. alpha1 antagonist b. alpha2 agonist c. beta1 agonist d. beta2 agonist e. antimuscarinic
  • 9. Basic Medical Science, March 2007 สหพันธสิตนักศึกษาแพทยแหงประเทศไทย National Council of Thai Medical Students (NCTMS) www.nctms.in.th -9- 63. มีอาการ แขนขาออนแรง right hemiparesis + motor aphasia เสียที่เสนเลือดใด a. ant. Cerebral a. right b. right middle cerebral a. c. right post. Cerebral a. d. left ant. Cerebral a. e. left middle cerebral a. 64. ทารกแรกเกิดมีอาการซีด บวมน้ําอยางมาก ตับมามโต ตรวจพบวารกมีขนาดใหญ ผิดปกติขณะคลอด มารดามี อาการครรภเปนพิษ ตรวจ VDRL ใหผลลบ ทารกถึงแกกรรม หลังคลอด 2 ชม. ผลการตรวจ Hb typing ไดผลเปนเชนใด a. alpha 4 b. beta 4 c. gamma 4 d. delta 4 e. zeta 4 65. ชายอายุ 50 ป เปนไตวายเรื้อรัง ตรวจรางกายพบซีด บวมขาทั้ง 2 ขาง คลํา ตับมามและตอมน้ําเหลืองไมพบ ตรวจเลือดพบ Hb 8 g/dl Hct 24% WBC 6000 N75% L25% platelet 200000 MCV 80 fL reticulocyte 1% creatinine 5 อาการซีดของผูปวยรายนี้ เกิดจากการขาด สารใดมากที่สุด a. Folic acid b. Erythropoietin c. Thyroid hormone d. Cyanocobalamine e. Colony stimulating factor 66. หญิง 15 ป มา ER ดวยอาการ หายใจและกลืนลําบาก แขนขาไมมีแรงเห็นภาพซอนหลังจากกินผัดกาดดองกระปอง มา 10 hr ลักษณะทางคลินิกดังกลาวมาจากอะไร a. ยับยั้งการหลั่ง Ach b. ยับยั้งการหลั่ง NE c. ยับยั้ง AchE d. กระตุน muscarinic Receptor e. กระตุน adrenergic receptor 67. ชายอายุ 50 ป เดินทางจาก กทม. ไป New York city ตอมามีอาการออนเพลีย นอนไมหลับ ระบบทางเดิน อาหารแปรปรวน อาการดังกลาวสัมพันธ กับ hormone จากโครงสรางตอไปนี้ a. anterior Pituitary b. posterior pituitary c. pineal gland d. hypothalamus e. adrenal gland 68.มารดาตั้งครรภ 28 สัปดาห คลอดทารกกอนกําหนด ทารกตัวเขียว ถูกวินิจฉัยวาเปน respiratory distress syndrome ให surfactance ไปแลว อาการดีขึ้น surfactant ที่ใหไป ออกฤทธิ์อยางไร a. เพิ่ม lung compliance b. ทําใหขนาดของถุงลมเทากัน 69. ผูปวยอายุ 70 ป มีอาการความจําเสื่อม แพทยวินิจฉัย วาเปน Alziemer สมองเสื่อมที่ใด a. cerebropontine b. cerebellum c. hippocampus d. medulla oblongata e. globus pulidus 70. ผูปวยเด็ก 5 ขวบ มีอาการไขต่ําๆ มา 1 เดือน พอเปน TB ทํา tuberculin test ในผูปวยเด็ก positive ถามวาผล ของการทํา tuberculin test เกิดจากปฎิกิริยาใด a. hypersensitivity type 1 b. hypersensitivity type 2 c. hypersensitivity type 3 d. delay type hypersensitivity e. none of above
  • 10. Basic Medical Science, March 2007 สหพันธสิตนักศึกษาแพทยแหงประเทศไทย National Council of Thai Medical Students (NCTMS) www.nctms.in.th -10- 71. ชายอายุ 30 ปมาดวยอาการหนังตาตก มีกลามเนื้อ ออนแรง ผูปวยมีอาการหนักมากขึ้นตอนเย็น มีตอม thymus โต กลไกการเกิดโรคเปนอยางไร a. immediate hypersensitivity b. delay type hypersensitivity c. immune complex hypersensitivity d. Ab-cytotoxic hypersensitivity e. Complement activation 72. เด็กทารกอายุ 2 วัน ชักเกร็งหลังแอนเปนพักๆ ตั้งแต แรกคลอด มารดาเปนชาวไทยภูเขา ไมไดฝากครรภ ครบ กําหนดคลอด หมอตําแยทําคลอดโดยใชไมรวกติดสาย สะดือ เด็กคนนี้เปนอะไร a. septicemia b. birth trauma c. hydrocephalus d. status epilepticus e. tetanus neonatorum 73. ชาย 50 ป มีตุมน้ําพุพอง ตามลําตัวและแขนขามา 1 เดือน ตรวจผิวหนังพบ vesicobullous eruption ทั่วทั้งตัว ทํา skin biopsy พบ acantholytic cell ที่ suprabasal bullae โครงสรางใดที่ผิดปกติ ( มีรูป ) a.microtubule b.desmosome c.keratin filament d.basement membrane e.keratohyaline granule 74. ชายอายุ 65 ป ไดรับการวินิจฉัยวาเปน alzheimer ถามวาอะไรเปนปจจัยที่สําคัญที่สุดที่ทําใหเซลลประสาท ถูกทําลาย ก. nerve growth hormone ลดลง ข. intracellular ca2+ ของเซลลประสาทลดลง ค. glycogenolysis ในเซลลลดลง ง. gluconeogenesis ในเซลลลดลง จ. ...... 75. หญิงอายุ 25 ป ตั้งครรภครบกําหนดคลอดแลวเกิด อาการเจ็บทองคลอด ผลที่เกิดขึ้นนี้เกิดจาก Hormone ใด 1) Prolactin 2) Oxytocin 3) Vasopressin 4) Catecholamine 5) Corticosteroid 76. คนไขหญิงอายุ 35 ป มีอุณหภูมิรางกายสูง มูกปาก มดลูกเปนลักษณะเหนียวขน ลักษณะฮอรโมนในขอใดที่ ถูกตอง 1. FSH สูง 2. testosterone สูง 3. estrogen ต่ํา 4. Progesterone สูง 5. Estrogen สูง 77. ชายอายุ 25ป ปสสาวะแสบขัด มีหนองไหลออกจาก ทอปสสาวะ นําdischargeไปยอมแกรมพบintracellular gram negative diplocci มีประวัติแพpenicillin ควรใหยา อะไรในการรักษา a. Imipenam b. Gentamicin c. Erythromycin d. Ciplofloxacin e. Bactrim 78.เด็ก 10 ขวบ ลมกระดูก humorus หัก สงผลใหเหยียด ขอมือกับนิ้วมือไมได มีสภาวะแทรกซอนที่ใด 1) median nerve 2) radial 3) Ulnar 4) medial cord of branchial plexus 5) lateral cord of branchial plexus 79. ชายอายุ 35 ป อดน้ําอดอาหารมาเปนเวลา 16 ชม. เพื่อเขารับการผาตัด พบ urine specific gravity = 1.025 hormone ใดในกระแสเลือดจะเพิ่มขึ้นมากที่สุด a.insulin b.oxytocin c.vasopressin d.glucocorticoid e.atrial natriuretic peptide
  • 11. Basic Medical Science, March 2007 สหพันธสิตนักศึกษาแพทยแหงประเทศไทย National Council of Thai Medical Students (NCTMS) www.nctms.in.th -11- 80. คนไข อยูในโรงพยาบาล อดขาวอดน้ํามา 16 ชั่วโมง ตรวจวา specific gravity 1.025 ถามวาฮอรโมนอะไรมีผล มากที่สุด ก. ADH ข. cortisol ค. insulin ง. glucagon จ. VIP 81. หญิงไทยคู อายุ 25 ป มีไข ปวดเมื่อยตามตัวมา 3 วัน ตอมามีแผลขึ้นที่ labia minora และ labia majora ลักษณะเปน painful red papules, vesicles และ coalescent ulcer ตรวจลักษณะทางจุลพยาธิวิทยา พบ intraepithelial vesicle พบ large cytoplasmic inclusion มี clear halo ลอรอบ และพบ multinucleated giant cell จงใหการวินิจฉัย 1. syphillis 2. acute eczema 3. gonococcal infection 4. herpes simplex infection 5. human papilloma virus infection 82. ชายอายุ 20 ป สุขภาพแข็งแรงดีมาตลอด กิน ยา Barbiturate เกินขนาดมา 1 ชั่วโมง ตรวจรางกายพบ RR เทากับ 12/min ถาตรวจ arterial blood gas จะพบอะไร ก. pH 7.5 ข. PaCO2 55 mmHg ค. PaO2 100 mmHg ง. Bicarbonate 18 mEq/L จ. Base excess +4 mEq/L 83. เด็กชายอายุ 15 ป อาศัยอยูในชุมชนแออัด ติดยาเสพ ติดมา 3 ป สถาบันใดมีบทบาทมากที่สุดในการปองกัน ปญหาสุขภาพจิตดังกลาว A. familial institute B. political institute C. religious institute D. Economic institute E. Education institute 84. ผูปวยชายเปนโรคตับ มี ascites กิน furosemide 1 เดือน ตอมาเกิดวิงเวียนเปนตะคริว ตรวจเลือดได Na 130, K 2.5 , pH 7.58 , pCO2 50 ความผิดปกติของสมดุลกรด เบสเกิดจากอะไร a- uncompensated respiratory alkalosis b- compensated respiratory alkalosis c- uncompensated metabolic alkalosis d- compensated metabolic alkalosis e- mixed respiratory and metabolic alkalosis 85. ผูปวยเจ็บเขาขางขวามา3วันขยับไมไดเจาะน้ําไขสัน หลังมาตรวจพบwbcเกิน50,000 pmnรอยละ80ผูปวย นาจะเปนโรคอะไร 1. osteoarthritis 2.rheumatoid arthritis 3. aspergilosis 4.tuberculoarthritis 86. หญิงอายุ 17 ป ไมมีประจําเดือน แตมี breast และ pubic hair เจริญปกติ คลําพบกอนที่ labia majora เคลื่อนที่ไปมาได การตรวจ ultrasound ไมพบ uterus ความผิดปกตินี้คือ A. testicular feminizing syndrome B. female psudohermaphrodite C. gonadal agenesis D. cryptorchism E. hypospadias 87. เด็กชายอายุ 5 ป มีจ้ําเขียวงายเมื่อถูกกระแทก มี เหงือกบวม และเลือดออกตามไรฟน ผลตรวจเลือดพบ platelet 200,000/ cu.mm. ลักษณะทางคลินิกดังกลาว เกิดจากความผิดปรกติใด a. Deamination of alanine b. Hydroxylation of praline c. Phosphorylation of serine d. Glycosylation of hemoglobin e. Carboxylation of glutamic acid
  • 12. Basic Medical Science, March 2007 สหพันธสิตนักศึกษาแพทยแหงประเทศไทย National Council of Thai Medical Students (NCTMS) www.nctms.in.th -12- 88. ชายอายุ 35 ป ถูกยิงตามรางกายหลายแหง ตรวจพบ PR 110/min BP 80/50 24ชม. ตอมา พบ BP 70/50-80/60 แมจะไดรับสารน้ําทดแทนและการรักษาเต็มที่ การที่ความ ดันโลหิตของผูปวยไมกลับคืนสูปกตินาจะมาจากสาเหตุใด a. Venous return เพิ่มขึ้น b. Central vasoconstriction c. Central venous pressure ต่ํา d. End diastolic pressure ลดลง e. Total peripheral resistance ลดลง 89. ชายอายุ 35 ป มีอาการออนแรงที่แขนขวา ตรวจ รางกายพบ spasticity และ hyperreflexia ของขาขวา แต มี fasciculation และ hyporeflexia ของแขนขวา อยาด ทราบวาอาการทางคลินิกดังกลาวเกิดจากพยาธิสภาพที่ใด a. Peripheral nerve b. Ventral root c. Spinal cord d. Brain stem e. Alpha motor neuron 90. ผูปวยชายไดรับการวินิจฉัยวาเปนไตวายเรื้อรัง Hb 8 HCT 24% platelet 200,000 WBC 6,000 creatinin 5 mg/dl ภาวะดังกลาวเกิดจากอะไรลดลง a. Folic acid b. Cyanocobalamin c. Erythropoietin d. Thyroid hormone e. Colony stimulating factor 91.ในการทํา systematic review เปรียบเทียบการใช amoxicillin กับ co-amoxiclav ในการรักษา sinusitis เพื่อ สนับสนุน การพิสูจน causal association ในขอใด a.temporalily b.reversibility c.consistency d.dose-effect e.biological plausibility 92. ผูปวยโรคแผลในกระเพาะอาหาร มีการหลั่งกรด จํานวนมากไมสามารถควบคุมดวยยาไดจึงทํา vagotomy เพื่อลดการหลั่งกรด vagotomy สงผลตอ parietal cell อยางไร a. Cl- channel b. H+ channel c. nicotinic receptor d. muscarinic receptor e. histamine receptor 93. ชายอายุ 30 ป ถูกงูกัดที่สนเทาซาย บวมเล็กนอย ไม เห็นตัวงู ตอมามีเลือดออกตามไรฟน และไตวายเฉียบพลัน ผูปวยรายนี้นาจะโดนงูชนิดใด 1. งูเหา 2. งูกะปะ 3. งูแมวเซา 4. งูทับสมิงคลา 5. งูเขียวหางไหม 94.ตรวจ ECG ของนักศึกษาแพทยคนหนึ่ง ผลในขอใด สามารถเกิดขึ้นไดปรกติ 1. PR interval ไมเทากัน 2. RR interval ไมเทากัน 3. บางชวง P wave หายไปได 4. P wave รูปรางไมเหมือนกัน 5. QRS complex กวางขึ้น หรือแคบลง 95. เด็กไมถาย meconium หลังคลอด ตอมาทองผูกและ ถายเหลวเปนบางครั้ง ตรวจพบไมมี ganglion cell ที่ submucosal layer เกิดจากความผิดปกติอะไร 1. migration 2. apoptosis 3. mutation 4. differentialtion 5. proliferation
  • 13. Basic Medical Science, March 2007 สหพันธสิตนักศึกษาแพทยแหงประเทศไทย National Council of Thai Medical Students (NCTMS) www.nctms.in.th -13- 96. เด็กทารกเพศชาย ถุงอัณฑะขางขวาใหญกวาขางซาย ลักษณะเปนถุงน้ําใส คลําเปนแนวเดียวกับลูกอัณฑะ คาด วาเปน a. hydrocele b. epispadias c. hypospadias d. inguinal hernia 97. squarmous cell CA ของ cervix เกี่ยวของกับเชื้อใด มากที่สุด a. Parvovirus b. Herpes c. EBV d. CMV e. HPV 98. ชาย 50 ป เหนื่อยงาย น้ําหนักลด x-ray พบกอน 3 cm. ที่ apex ของปอดขวา ผาตัดกอนออกมา ดู histo พบดัง ภาพ คาดวาผูปวยเปน a. Aspergillosis b. Tuberculosis c. Lung abscess d. Cryptococcosis d. Bronchial carcinoma 99. เด็กชายอายุ 5 ป มีไขสูง ซึม อาเจียน ตรวจพบ stiffness of neck ผลการตรวจ CSF พบ WBC 1,200 cell/cu.mm. N 90% MN 10% protein 150 μg/dl sugar 20 μg/dl(blood sugar 100) เชื้อกอโรคใดมีโอกาสเปนไป ไดมากที่สุด a. Enterovirus b. Neagleria floweli c. Neisseria meningitidis d. Cryptococcus neoformans e. Mycobacterium tuberculosis 100. เด็กชายอายุ 7 ป 3 สัปดาหกอนมีไข 2 วันกอนเขา โรงพยาบาล มีอาการเทาบวม กดไมบุม ตรวจรางกายพบ BP170/100 protein 2+ WBC 10-20/HPF พบ numerous dysmorphic RBC พบ RBC cast 1-2/HPF อยากทราบวามี pathology ที่บริเวณใด a. Glomerulus b. Proximal tubule c. Loop diuretic d. Distal tubule e. Interstitium 101. ชายอายุ 35 ป ไมเคยออกกําลังกายมากอน พอเตน แอโรบิกครั้งแรก เสียเหงื่อเปนจํานวนมาก ตรวจ BP กอน และหลังปรกติ อยากทราบวารางกายใชระบบ renin- angiotensin system ในการรักษา BP อยางไร a. Vasodilation b. ↑ renal blood flow c. ↑ sodium reabsorption d. ↑ GFR e. ↓ angiotensin 102. ผูหญิงวัยกลางคน เปนไขต่ํามา 2 วัน แลวปวดหลัง บริเวณลางซาย แลวลามมาที่ขาหนีบซายอยางฉับพลัน x-ray พบ ureteral calculi นาจะเกี่ยวกับการติดเชื้อใด a. Esterichia coli b. Protius mirabilis c. Enterococus faecialis d. Krebsiella pneumonia e. Staphylococcus saprophyticus
  • 14. Basic Medical Science, March 2007 สหพันธสิตนักศึกษาแพทยแหงประเทศไทย National Council of Thai Medical Students (NCTMS) www.nctms.in.th -14- 103. ชายอายุ 70 ป สูบบุหรี่จัดมา 40 ป เหนื่อยงาย หอบ เปนครั้งคราว ตรวจ x-ray พบวาเปน emphysema คาใด ของ lung function test ลดลง a. Airway resistance b. Lung compliance c. Intrapleural pressure d. FVC e. FEV1 104. ยา X ให ผลเปน ½ เทาของ max ที่ 5 mgใน 1 hr ยา Y ให ผลเปน ½ เทาของ max ที่ 25 mgใน 2 ชั่วโมง ยา X มีผลเปนกี่เทาของยา Y a. 5 b. 10 c. 25 d. 40 e. 50 105. หญิงอายุ 35 ป ตั้งครรภ 32 สัปดาห ตรวจ ultrasonography พบวาเด็กในครรภมีไตซายขางเดียว และมีปริมาณน้ําคร่ํานอย ความผิดปรกติดังกลาวนาจะ เกิดจากสาเหตุใด a. Oligohydramnios b. Polycystic kidney c. Ureteric bud duplication d. Degeneration of mesonephros e. Absence of metanephrogenic tissue 106. เด็กชายอายุ 5 ป มีจ้ําเขียวงายเมื่อถูกกระแทก มี เหงือกบวม และเลือดออกตามไรฟน ผลตรวจเลือดพบ platelet 200,000/ cu.mm. ลักษณะทางคลินิกดังกลาว เกิดจากความผิดปรกติใด 1. Deamination of alanine 2. Hydroxylation of praline 3. Phosphorylation of serine 4. Glycosylation of hemoglobin 5. Carboxylation of glutamic acid 107. (A).รูป. ถามวาเปนกระบวนการใด { normal cell →chromatin dense → nucleus แตกเปน bleb → cell อื่นมาเก็บกิน } a. necrosis b. karyolysis c. apoptosis d. pinocytosis e. endocytosis 108. การสํารวจ CA breast stage 1,2,3,4 พบผูปวยรอย ละ 20,40,60,80 ตัวแปรการศึกษานี้เปนแบบใด (ขอนี้มี 4 ตัวเลือก) a. Ratio b. Interval c. Nominal d. Ordinal 109. ชายอายุ 20 ป ขณะเลนฟุตบอลหายใจเร็วและแรง ขึ้น การเพิ่มของปจจัยใดทําใหรางกายไดรับอากาศใหม มากที่สุดขณะเลนฟุตบอล a. Tidal volumn b. RR c. Minute ventilation d. Dead space ventilation e. Inspire reserve volumn(IRV) 120. ชาย 30 ป ไข ไอ หอบ เหนื่อย 2 วัน ตรวจรางกายพบ fine crepitation ที่ปอดลางขวา แสดงถึงการอักเสบ บริเวณใด a. Alveoli b. Trachea c. Main bronchus d. Lobar bronchus e. Segmented bronchus
  • 15. Basic Medical Science, March 2007 สหพันธสิตนักศึกษาแพทยแหงประเทศไทย National Council of Thai Medical Students (NCTMS) www.nctms.in.th -15- 121. ชาย 65 ป เปน COPD+bronchietasis มีประวัติสูบ บุหรี่เยอะ เสมหะมาก หายใจเหนื่อยแมพัก คาทางสรีระ วิทยาใดสัมพันธกับผูปวย a. FEV1/FVC ปรกติ b. Airways resistance สูงขึ้น c. Residual volumn ลดลง d. Lung compliance ลดลง e. Lung capacity เพิ่มขึ้น 122. หญิง 30 ป ทานอาหารทะเล เกิดผื่น erythrematous plaque cell ใดเปน mediator หลัก a. Basophil b. Eosinophil c. Macrophage d. Neutrophil e. Lymphocyte 123.ชายอายุ 30ป เปนthalassemiaตองไดรับเลือดแตเด็ก มีผิวคล้ํา serum glucose 140 พยาธิสภาพของตับออน เปนอยางไร a. Insulinitis b. Pancreatitis c. มี amyloid สะสม d. Hemochronatosis 124. ชาย 60 ป เปนเบาหวานมา 20 ป มีแผลที่เทา ฉีด Stem cell แลวพบวาแผลหายและเล็กลง stem cell คือ อะไร A. plasticity B. maturation C. induction D. migration E. cell differentiate 125. เด็กชายอายุ 10 ขวบ มีไข 2 วัน ตัวเหลือง ซีด Hct 25% Hb 7 WBC 5000 ( N 65% L 35% M 5% ) Plt 250000 ตรวจพบHeinzbodyเกิดจากกกระบวนการใด a. Precipitate ของ Ribosome b. Precipitate ของ Hemoglobin c. Iron inclusion d. การแตกของ Nucleus e. Aggregation ของ ER 126. เพศชาย 70 ป ออนเพลียมากมา 7 วัน มีประวัติ ปวดทองและอุจจาระดํามา 3 เดือน Hct 23 WBC 3200 ( N 30% L 70% ) Plt 70000 นาจะเกิดจาก สาเหตุใด a. เสียเลือดเรือรัง b. Autoantibody c. ขาดเซลลตั้งตนในไขกระดูก d. มีการใชเกร็ดเลือดเปนจํานวนมาก e. ขาดสารอาหารในการสราง RBC 127. เด็กมีผิวกายซีด สีผมจาง ขนออนเปนสีขาว ตาดํา เปนสีแดง มีพอและแมปกติ นาจะเกิดจากความผิดปกติ ของอะไร a. ectoderm b. mesoderm c. endoderm d. neural crest e .neural tube 128. หญิงแตงงานมา 3 เดือนหลังจากนั้น 1 เดือน ประจําเดือนหายไป ควรตรวจ hormoneใดเพื่อดูการ ตั้งครรภ a. estrogen b. progesterone c. FSH d. LH e. HCG
  • 16. Basic Medical Science, March 2007 สหพันธสิตนักศึกษาแพทยแหงประเทศไทย National Council of Thai Medical Students (NCTMS) www.nctms.in.th -16- 129. pt หญิงอายุ 40 ปมีกอนที่คอ ตอมไทรอยด PR 70 ไมมีอาการใจสั่น มือสั่น พบขุยสีขาวที่ผิวกอน นาจะเปนอะไร a. follicular adenoma b. papillary carcinoma c. follicular carcinoma d. medulary carcinoma e. anaplastic carcinoma 130. N-acetylcysteinที่ใชในการแกไขภาวะ paracetalmol toxicity มีกลไกในอารออกฤทธิ์เกี่ยวของ กับกลไกใด ของ liver a. N-acetylation b. N-hydroxylation c. sulfer conjugate d. glutathione conjugate e. glucolonide conjugate 131.หญิงอายุ 30ตั้งครรภแรกตรวจเลือดพบ Anti-HBsAg ได positive สวน Anti-HBcAb และ Anti-HBsAb ได negative 6 เดือนตอมา ตรวจAnti-HBsAg และ Anti- HBcAb ได positive สวน Anti-HBsAbได negative จงแปลผลตรวจ Serology a. เคยไดรับวัคซีน b. อยูในชวงพาหะ c. อยูในชวงระยะฟกตัว d. ติดเชื้ออักเสบเฉียบพลัน e. เคยไดรับเชื้อและมีภูมิตานทานอยูแลว 132. Patient มีอาการแนนหนาอกดานซายมา 1 สัปดาห ตรวจรางกายพบ dullness, breath sound ขางซายนอย กวาขางขวา และมี trachea เบนไปทางขวา นาจะเปนโรค ใด a. pneumothorax b. pleural effusion c. lobar pneumonia d. lung atelectasis e. rib fracture 133. ชายอายุ 20 มีอาการอาเจียน ปวดทองใตชายโครง ดานขวา ราวไปถึงตนแขนขวา ไดรับการวินิจฉัยวาเปน acute hepatitis ถามวา การตรวจในขอใดบงชี้วา มีอาการ รุนแรงที่สุด a. Ammonia > 80 b. AST(SGOT) > 2000 c. Blood sugar > 160 d. Prothrombin > 38 e. Alkaline phosphatase > 300 134. ผูชาย 50 ป เหนื่อยหอบ หนาบวม มี engorge jugular venous , dilate superficial vein on chest มี hilar lymph node โต ไมมีรอยโรคอื่นๆ จากการ X-ray นาจะเปนโรคใด a. Tuberculosis b. Aspergilosis c. Lung abcess d. Brocheal adenoma e. Brochogenic CA
  • 17. Basic Medical Science, March 2007 สหพันธสิตนักศึกษาแพทยแหงประเทศไทย National Council of Thai Medical Students (NCTMS) www.nctms.in.th -17- 135. เด็กชายอายุ 5 ป ไขสูงมา3วัน ชัก ซึม Stifness of neck ตรวจรางกายพบวาไมรูสึกตัว ตรวจ CFS โดยการ ยอม Gram พบ Gram negative diplococci ควรใหการ ดูแลใกลชิดผูปวยยังไง a.ใหยาปฏิชีวนะทุกคน b.ใหยาปฏิชีวนะทุกคน และใหวัคซีนเพิ่มกับญาติ ผูปวย c.ใหยาปฏิชีวนะทุกคน และใหวัคซีนเพิ่มกับ บุคลากร ทางการแพทย d.ใหวัคซีนกับทุกคน และใหยาปฏิชีวนะกับญาติ ผูปวย e.ใหวัคซีนกับทุกคน และใหยาปฏิชีวนะกับบุคลากร ทางการแพทย 136.ผูชายประสบอุบัติเหตุทางรถยนตเปนผลใหมีการ เปลี่ยนแปลง RR (เพิ่มขึ้น), BP 200/100 BPเพิ่มขึ้นเพราะ เหตุใด a. Preload เพิ่ม b. afterloadเพิ่ม c. Sympathetictone ลด d. มี vasoconstriction of peripheral vessels 137. ผูหญิงอายุ 60 ปเปน เบาหวานมี BMI=30 ถามวา ควรจะใหยาใดในการรักษาผูปวยคนนี้ a.Metformin b.Acarbose c.Meglitinide d.Glitazone e.Glipizide 138. คนขึ้นที่สูง 800 ฟุตเหนือน้ําทะเล มีการหายใจเร็วขึ้น เกิดจากการกระตุนกลไกใด a. J-receptor b. Irritant receptor c. peripheral chemoreceptor d. central chemoreceptor 139. Cushing syndrome จากยาหมอ ตรวจเลือดจะพบ อะไร a. Na+ นอย b. K+ นอย c. Ca+ มาก d. Cl- มาก e. metabolic alkalosis 140. สารใดยับยั้ง apoptosis A. P53 protein B. bcl-2 C. IL-6 D. tyrosine kinase E. TNF 141. ชายอายุ 40ป ปวดที่ first metatarsophalangeal joint , serum uric acid สูง ขอใดผิด a. เปนโรค gout b. เกิดจากความผิดปกติใน pyrimidine salvage pathway c. กินอาหารที่มี nucleic acid สูง d. มีภาวะ acidosis สูง 142.เด็กอายุ 10 ป ลมแลวมี fracture ที่ humorus ถามวา มีผลขางเคียงตอ เสนประสาทใด A. ulna nerve B. median nerve C. radial nerve D. Internal cord of brachial plexus E. External cord of brachial plexus 143. ผูปวยไตวายเรื้อรัง K= 6, Na = 135 , Cl ปกติ ถาม วาควรใหยาอะไร 1. Spinoloractone 2. Priqnerene 3. Amiloride 4. Furosemide 5. Eplereone
  • 18. Basic Medical Science, March 2007 สหพันธสิตนักศึกษาแพทยแหงประเทศไทย National Council of Thai Medical Students (NCTMS) www.nctms.in.th -18- 144. ชายอายุ ? ป มีอาการ weak ที่ distal limb แลว อาการ progress ที่ proximal เจาะ CSF ได wbc 3, protein 250, sugar 80, เขาไดกับโรคใด A. Toxic neuropathy B. Myasthenia gravis C. Reverse D. Guillain-Barr syndrome C. Brown-sequard 145. ผูปวนเดินเกร็ง หนาไรอารมณ มีresting tremor Metabolism สวนใดของระบบประสาทที่นอยที่สุด A. Cerebellum B. Hippocampus C. basal ganglia D. anterior horn cell E. mammillary body 146. ชายอายุ 35 ป มีอาการออนแรงที่แขนขวา ตรวจ รางกายพบ spasticity และ hyperreflexia ของขาขวา แต มี fasciculation และ hyporeflexia ของแขนขวา อาการ ทางคลินิกดังกลาว นาจะเกิดจากพยาธิสภาพที่ใด ก. peripheral nerve ข. ventral root ค. spinal cord ง. brain stem จ. alpha motor neuron 147. หญิง อายุ 20 ป เปน จิตเภท รักษาดวย Haloperidol แลวมีอาการลิ้นแข็ง เกิดจากกลไกใด A. inhibit dopaminergic B. inhibit cholinergic C. inhibit serotonin D. กระตุน dopaminergic E. กระตุน cholinergic 148. ผูปวยเปนโรค B-thalassemia HbE disease แตงงานกับคนที่เปน HbE homozygote โอกาสมีลูกเปน โรค thalassemia เทากับขอใด A. 1 B. 3/4 C. 1/2 D. 1/4 E. 0 149. ผูหญิงแก มีอาการหลงลืม แพทย Dx Alzheimer มี ความผิดปกติทีสมอง สวนใด A. crus cerebri B. cerebellum C. hippocampus D. Globus pallidus 150. ทารกชายอายุ 1 วัน มีอาการหายใจลําบาก ตรวจ รางกายพบหนาทองแฟบ ฟงไดเสียง peristaltic sound ที่ ชองอกดานขวา หัวใจและtrachea ถูกดันมาอยูทางดาน ขวา ลักษณะทางคลินิกดังกลวเกิดจากความผิดปกติของ โครงสรางใด 1. later body wall 2. septum tranversum 3. costodiaphragmatic recess 4. pluroperitoneal membrane 5. dorsal mesentery of esophagus 151. ชายอายุ 40 ป ไดรับยาเคมีบําบัดรักษามะเร็งเม็ด เลือดขาว หลังจากนั้นมีไข จํานวนเม็ดเลือดขาวลดลง ไดรับยาปฏิชีวนะเปนเวลานาน ระหวางที่ไดรับยา ปฏิชีวนะมีอาการปวดทอง อุจจาระรวง ไมพบเม็ดเลือด แดงและ เม็ดเลือดขาว ผลการเพาะเชื้อจากอุจจาระไมพบ normal flora เชื้อที่เปนสาเหตุนาจะเปนเชื้อใด 1. Clostridium difficile 2. Shigella dysenteriae 3. Salmonella enterica 4. Yernisia enterocolita 5. Plesiomonas shigelloides 152. Pathway ที่ใหพลังงานแก RBC a. Urea cycle b. Kreb’s Cycle c. Beta oxidation d. Lactose fermentation e. Glycolysis
  • 19. Basic Medical Science, March 2007 สหพันธสิตนักศึกษาแพทยแหงประเทศไทย National Council of Thai Medical Students (NCTMS) www.nctms.in.th -19- 153. ผูปวยหญิงอายุ 45 ป เพลียซีด อาเจียนมาก และ บวมมา 2 สัปดาห เมื่อ 2 ป กอนเคยมีอาการบวม ทั้งตัว ตรวจรางกายพบ BP 180/110 ,lung sound: crepitation ที่ปอดสวนลาง บวมที่หนาขาทั้ง 2 ขาง urinalysis พบ specific gravity 1.01 albumin 2+ wbc 10/lpf rbc 20/lpf broad waxy cast 1-2 hpf คนไขนาจะเปน a. Chronic renal failure b. Renal cell carcinoma c. Acute tubular necrosis d. Renal tubular acidosis e. Rapid progressive glomerulus nephritis 154. ผูปวยชายอายุ 60 ป มีอาการตัวเหลืองตาเหลือง คัน ทั่วตัว และอุจจาระซีด ตรวจรางกายคลําตับได 1 ซม. ใต ชายโครงขวา ผูปวยนาจะมีระดับของสารใดสูงขึ้น a. stercobillin b. serum albumin c. urine urobillinogen d. plasma prothrombin e. serum direct bilirubin 155. ผูปวยหญิงโสดอายุ 30 ป ไมมีประจําเดือนมานาน มีน้ํานมตลอด 8 เดือน ที่ผานมาตรวจ urine pregnancy test ไดผลลบ ผูปวยนาจะมีระดับสารใดในเลือดสูง a.prolactin b.estradiol c.progesterone d.leutinizing hormone e.follicular stimulating hormone 156. หญิงมีกอนที่ขอเทา แผลมีรูเปดหลายรู บีบแลวมี หนองปนเม็ดเล็กๆ สีเหลือง ยอม modified acid fast ใหผลเปนลบ 1. Nocardia asteroids 2. Penicillium manesfii 3. Clostidium perfringen 4. Mycobacterium chelone 5. Streptomyces spp 157. หญิงหาบเร อายุ 30 ป ไมมีโรคประจําตัว แตงงาน มา 5 ป มาพบแพทยดวยเรื่องไมมีบุตร มีประวัติปสสาวะ แสบขัดบอย และตกขาว ซื้อยากินเอง สามีมีประวัติเที่ยว หญิงบริการบอยโดยไมสวมถุงยางอนามัย ผลการตรวจ รางกายอสุจิของสามี analysis อยูในเกณฑปกติ ปจจัยใด ที่ทําใหผูหญิงคนนี้ไมมีบุตร a. อาชีพ b. เศรษฐกิจ c. ซื้อยากินเอง d. สามีสําสอน e. มีโรคติดตอทางเพศสัมพันธบอย 158. หญิง 25 ป ตั้งครรภ 40 สัปดาห เจ็บทองพรอมคลอด แตไมมีแรงเบงลูก จะใหยาชนิดใดจะไดคลอดบุตรงาย a. Oxytocin b. Ritrodine c. Ergotarmine d. Vasopressin e. Bromocriptine 159. ชายไทยอายุ 50 ป มีอาการเจ็บหนาอก แพทยให การวินิจฉัยวาเปน acute myocardial infarct จึงใหยา ละลายเลือดเพื่อรักษา ถามวายานี้มี mechanism เพิ่ม activity ของสารใด a. plasmin b. Protein C c. AntiThrombin d. Prostacyclin e. Phospholipid 160. กลไกใดของเกล็ดเลือดเกิดเปนอันดับแรกใน กระบวนการหามเลือด a. เกิดการหลั่ง ADP b. มีการปลอย Prostacycline c. Activation ของ COX enz d. Activation ของ thrombomodulin e. Activation ของ glycoprotein และ VWF
  • 20. Basic Medical Science, March 2007 สหพันธสิตนักศึกษาแพทยแหงประเทศไทย National Council of Thai Medical Students (NCTMS) www.nctms.in.th -20- 161. หญิงอายุ 20 ป กินยานอนหลับไมทราบขนาด ไมมี สติ (unconscious) หายใจชาและลึก ประมาณ 8 ครั้ง/ นาที วัด arterial blood gas : pH = 7.07 paO2 = 32 paCO2 = 88 HCO=24 แพทยไดใชเครื่องชวยหายใจ เครื่องชวยหายใจมีประโยชนตอคนไขรายนี้ในดานใดมาก ที่สุด a. เพิ่ม oxygen ในเลือด b. เพิ่ม minute ventilation c. เพิ่ม diffusing capacity d. ลด resistance e. ลด intrapulmonary shunt 162. ชายอายุ 40 ป อาชีพแกะสลักชื่อบนหินออน ไอและ หอบเหนื่อยมา 2 ป ประวัติสัมพันธสารพิษ มา 4 ป พยาธิ สภาพที่เกิดขึ้นจะเกิดขึ้นที่ใด a. Nose b. nasopharynx c. Trachea d.bronchus e. alveoli 163. ชายอายุ 15 ป มีกลิ่นตัวทําใหขาดความมั่นใจ ปญหาดังกลาวเกิดจากโครงสรางใดของผิวหนัง ก. epidermis ข. hair follicle ค. apocrine gland ง. sebaceous gland จ. arrector pilli muscle 164. การศึกษาเปรียบเทียบประวัติการไดรับวัคซีนปองกัน โรคหัดในวัยเด็กระหวางนักเรียนที่เปนโรคและไมเปนโรค หัด พบวา odds ratio ของการเปนโรคหัด ในกลุมที่ไดรับ วัคซีนเปรียบเทียบกับกลุมที่ไมเคยไดรับวัคซีน = 0.05 165. ประสิทธิผลของวัคซีนจากการศึกษาดังกลาว = รอย ละเทาไร a. 0.05 b. 0.95 c. 5 d. 95 e. 99.95 166. ครูพบนักศึกษาชายหญิงกอดจูบกันที่ปายรถเมล จึง เขาไปตักเตือนวาเปนพฤติกรรมที่ไมเหมาะสม ครูใชเกณฑ ใดเปนเครื่องมือในการตัดสินพฤติกรรมนักเรียน a. faith b. value c. norms d. belief e. attitude 167. หญิงอายุ 50 ป มีอาการเหนื่อยงายมา 1 เดือน มี อาการชามือชาเทา มีประวัติรับประทานอาหารมังสวิรัติมา 2 ป ตรวจรางกายพบวาซีด จากอาการทางคลินิก แสดงวา ขาดวิตามินใด a. Retinol b. Thiamine c. Riboflavin d. Cobalamine e. Niacin 168. ชายอายุ 60 ป มีอาการอุจจาระรวงสลับทองผูกมา 3 week adenocarcinoma sigmoid colon แพทยไดทํา รักษา โดยการผาตัดสวนที่เปนมะเร็งออกและทําเคมีบําบัด แลว follow up ดวยการดู tumor marker ชนิดใด a. CA-125 b. CA 15-3 c. alpha fetoprotein d. lactic dehydrogenase e. Carcinoembriogenic antigen 169. เด็กอายุ 1 ป มีความบกพรองของ hydrolytic enz. มี ผลตอ organelle ใด a. acrosome b. lysosome c. ribosome d. microsome e. peroxisome
  • 21. Basic Medical Science, March 2007 สหพันธสิตนักศึกษาแพทยแหงประเทศไทย National Council of Thai Medical Students (NCTMS) www.nctms.in.th -21- 170. อัตราการไหลเวียนเลือด 100 ml/min ถาหลอดเลือด มีรัศมีลดลง 50% โดยความดันเทาเดิม จะมีอัตรา ไหลเวียนเลือดเทาไร a. 6.25 b. 25 c. 50 d..... e..... 171. เด็ก 8 ป ลักษณะเตี้ยขาโกง Ca 7.5 mg/dl, phosphate 2.8mg/dl, alkaline phosphatase 300microI/L อาการของเด็กเกิดจากการขาดสารอะไร a. Ca b. Vit D c. Vit C d. Calcitonin e. PTH 172. เด็กอายุ10ป มีอาการอานเพลีย occult blood positive ไดทําstool exam ผลปรากฏดังภาพ ควรแนะนําใหผูปวย a. ไมเดินเทาเปลา b. ไมกินผักสด c. ไมกินปลาราดิบ d. ไมกินหมูสุกดิบ e. ลางมือทุกครั้งหลังออกจากหองน้ํา 173. หญิงอายุ50ป เลือดกะปริบกะปรอยทางชองคลอดมา 1 week ตรวจภายในพบมดลูกโตขึ้นเล็กนอย จากการ ตรวจชิ้นเนื้อพบ proliferative ของendometrial gland อยู ชิดกันหนาแนน, cellบุผนังมดลูกมี Nucleus ใหญ, ติดสี เขม, Nucleolusชัดเจน พบ mitotic figure all ปรากฏการณที่พบเกิดจากอะไร 1. endometriosis 2. HPV infection 3. Gonococcal cervicitis 4. Infertility 5. Low-socio-economic status 174. หญิงอายุ 40 ป ทําหนาที่เปนเจาหนาที่ฝายบัญชีมีคา BMI 26.5 2 ปที่แลวไดรับการผาตัดรังไขออกทั้ง 2 ขาง ตอนนี้เปน Osteoporosis อยูถามวา อะไรที่เปนปจจัยที่ทํา ใหคนนี้เปน osteoporosis a. อายุ b. เพศ c. ดัชนีมวลการ d. อาชีพ e. ผาตัด ovary ออก 175. ชาย 45 ป ปวดเปนๆหายๆ ที่ MTP jt. ของนิ้วหัวแม เทาขางขวา uric 8.2 mg/dl ผิดปกติ metabolism ของ สารใด a. purine b. protein c. glucose d. cholesterol e. pyrimidine 176. ชาย 40 ป มีอาการปวดกลามเนื้อ กลามเนื้อคลายตัว ชา ขาดสารอะไร a. Ca b. phosphate c. Creatinine phosphate d. ADP e. ATP
  • 22. Basic Medical Science, March 2007 สหพันธสิตนักศึกษาแพทยแหงประเทศไทย National Council of Thai Medical Students (NCTMS) www.nctms.in.th -22- 177. หญิงอายุ 30 ป ญาติพามาปรึกษาเนื่องจากเปลี่ยน งานบอย ใหเหตุผลวาเพื่อนรวมงานและหัวหนาคอยแกลง อยูในหองตลอดเวลา บางครั้งพูดคนเดียว โมโหญาติคิดวา ญาติคอยแกลงเปนอาการใด a. delirium b. schizophrenia c. delusional disorder d. Major depressional disorder e. Substance related disorder 178. เด็กหญิงอายุ 2 ป มีกระจุกบน (hair patch) บริเวณ sacral region ตรวจรางกาย ไมพบความผิดปกติในระบบ ประสาท อาการแสดงนี้มักพบรวมกับอาการใด 1. Rachischisis 2. Spina bifida cystica 3. Spina bufida occulta 4. Spina bifida with myeloschisis 5. Spina bifida with meningomyelocele 179. อายุ27ป กิน carbamet แลวมีอาการปวดทอง, muscle fasciculation, น้ําลายยืด, เหงื่อออก, PR 60 beat/min หมอให Atropine เปนยารักษาแลวจะยังคงมี อาการใดคงอยู 1. ปวดทอง 2. Muscle fasciculation 3. น้ําลายยืด 4. เหงื่อออก 5. 60 beat/min 180. ชายอายุ 20 ปสุขภาพแข็งแรง หลังจากบริจาคเลือด ไป 1 unit เกือบหมดสติเปนลม BP 80/50 , PR 50/min นาจะเกิดจากสาเหตุใด a. cardiogenic shock b. hypovolumic shock c. vasovagal attack d. orthostatic hypotention e. stroke adams 181. ชายอายุ 50 ป มีแผลที่หลัง เปนผื่นขาววงกลม ขอบ แดงนูน ขูดบริเวณแผลมาตรวจ KOH preparation พบเชื้อ hyaline septate hyphae ใชยาใด a. Fluconazole b. Clotimazole c. Amphotericin B d. Bemzyl benzoate e. Sodium thiosulfate 182. ลําดับเบสของผูปวยที่มี mutatuion เปน ..ATGCCAAA.. หากตองการหาDNA probe ควรใชลําดับ เบสใด ก. UUUGGCAU ข. UACGGUUU ค. TTTGGCAT ง. TACGGTTT จ. TTTGGUAT 183. ในการทดลองยา X ที่มีผลตอ telomerase เพื่อเปน ยาตานCA ยาควรยับยั้งกระบวนการใดของ CA ก. DNA stability ข. DNA replication ค. DNA degradation ง. Gene rearrangement จ. Spindle fiber attachment 184. หญิงอายุ 24ป อยุระหวางใหนมบุตร พบวามีกอนที RT outer quadrant ที่เตานมขวา กดเจ็บ รอบๆกอนรอน และมีสีแดง กินยาแลวอาการดีขึ้น ตอมาบริเวณที่เคยเปน กอนกลายเปนผิวหนังที่บุมลงไป สิ่งที่เกิดขึ้นเปนพยาธิ สภาพแบบใด a. fibroadenoma b. fibrocystic change c. traumatic fat necrosis d. breast abscess with scar e. infiltrating mammary adenocarcinoma
  • 23. Basic Medical Science, March 2007 สหพันธสิตนักศึกษาแพทยแหงประเทศไทย National Council of Thai Medical Students (NCTMS) www.nctms.in.th -23- 185. ผูปวยหญิงอายุ 35 ป ตัดไขและปกมดลูกออกทั้งสอง ขางเนื่องจากมีเนื้องอก แตผูปวยกลัววาจะมีภาวะกระดูก พรุน ถามวายาหรือสารใดที่เปน single treatment ของ กระดูกพรุน 1. Ca 2. Vit D 3. Prednisolone 4. Progesterone 5. Estrogen 186. เด็กผูหญิงอายุ 6 เดือน แมพบวามีน้ําซึมออกมาจาก สะดือ นาจะเปนโรคใด ก. Urachal fistula ข. Omphalocele ค. gastrichisis ง. neuroblastoma 187. คนไขประสบอุบัติเหตุ ทางรถยนต มีปสสาวะออกมา 5000 ml urine graviy 1.015 คนไขคนนี้มีความผิดปกติที่ ใด ก. ……. ข. การ reabsorb sugar ค. การขับ... ง. การ reabsorb water ที่ collecting duct จ. การ reabsorb Na ที่ PCT 188. หญิงอายุ 50 ป ทานขาวมันไก 2 จาน และฟกทอง แกงบวช 1 ถวย หลังจากนั้นรูสึกทองอืด และหลังจากนั้นก็ รูสึกอิ่ม ถามวา hormone จาก GI อะไรที่ทําใหเกิด เหตุการณนี้ ก. Motilin ข. Gastrin ค. Secretin ง. CCK จ. VIP 189. หญิงอายุ 30 ป อุจจาระรวงเปนน้ําสลับกับอาเจียน เปนเวลา 6 ชั่วโมง กอนหนานี้ทานขาวคลุกกะป คิดวาเชื้อ อะไรเปนสาเหตู A. E.coli B. Virio chlorella C. Shigella D. Bacillus cereus E. Aeromonas hydrophilia 190. สามีภรรยา มีลูกเปน Hb H เจาะเลือดสามีภรรยา Hct Hb MCV Reticulocyte สามี 38 13 72 1% ภรรยา 39 13 87 0.5% Genotype ใดถูกตองที่สุด a. สามี αα/-- b. ภรรยา α-/α- c. ภรรยา αα/αα d. สามี βoβt e. ภรรยา βoβo 191. เด็กเปน giloma ไดรับรังสีรักษา ตอมากอนไดยุบลง นิดเดียว เปนเพราะอะไร a. inactivation p53 b. inactivation BCl-2 c. Inactivation MPL- d. Activation retinoblastoma e. Activation BCL 192. กินมังสวิรัต 2 ป ขาด Vit อะไร 1. thiamine 2. folate 3. Riboflavin 4. B12 5. niacin
  • 24. Basic Medical Science, March 2007 สหพันธสิตนักศึกษาแพทยแหงประเทศไทย National Council of Thai Medical Students (NCTMS) www.nctms.in.th -24- 193. ผูปวยชายอายุ 30 ป มีประวัติปวดทองและถายดํา มา 3 เดือน คลําไมพบตับ มาม ตอมน้ําเหลืองโต ตรวจ พบ Hct 23% WBC 3,200 Neu 30% Lymphocyte 70% Platelet 50,000 MCV 89 ขอใดอธิบยภาวะโรคของผูปวย ไดถูกตอง 1. การสราง RBC นอยลง 2. มามทําลาย RBC มากไป 3. มี Ab มาทําลาย RBC มาก 4. ขาดเซลลตนกําเนิดในการสราง 5. ขาดสารอาหารในการสราง 194. เด็กชายอายุ 5 ป เปน Hemophilia A ใชอะไรในการ ชดเชย coagulation 1. Aged plasma 2. Cryoprecipitate 3. Fresh frozen plasma 4. Platelet 5. Fresh whole blood 195. ศูนยมะเร็งแหงหนึ่งศึกษาเกี่ยวกับความรุนแรงของ มะเร็งเตานม มี 4 ระยะคิดเปอรเซ็นต ดังนี้ 20,35,35,10 การศึกษานี้มีลักษณะขอมูลแบบใด 1. Interval 2. Ordinal 3. Ratio 4. Nominal 196. คนไขปวดทองบริเวณรอบสะดือ จากนั้นยายมาที่ ทองนอยดานขวา หมอ diag วาเปนไสติ่งอักเสบ คนไขจะ กดเจ็บบริเวณใด a. ¼ ของระยะทางจาก ASIS ไปสะดือ b. ¼ ของระยะทางจาก AIIS ไปสะดือ c. 1/3 ของระยะทางจาก ASIS ไปสะดือ d. 1/3 ของระยะทางจาก AIIS ไปสะดือ e. ¼ ระยะทางจาก iliac crest ไปสะดือ 197. ทารก 3 วันคลอดกอนกําหนด หนัก 2,300 g หายใจ เร็ว ไมเขียว ฟงเสียง murmur ไดที่ left 2nd intercostals space เมื่อให indomethacin เสียง murmur หายไป ทารกนี้มีโรคหัวใจคืออะไร a. Aortic stenosis b. Pulmunic stenosis c. ASD d. VSD e. PDA 198. เด็กเปนฝที่ขา เมื่อ Phagocytosis เขาไป ใช enz ใด ใน O2-dependent mechanism ใน neutrophil 1. Lysozyme 2. Defensin 3. Hydrolaze 4. Lactoferrin 199. หญิงอายุ 35 ป ซีด ออนแรง เหนื่อยงาย เปน peptic ulcer ถายอุจจาระดํา ตรวจรางกายพบซีดปาน กลาง มีไข เปนๆหายๆ ตาเหลือง ตับไมโต แต มามโต ตรวจเลือดพบ Hb 8 g/dL Hct 30 Reticulocyte 8% Platelet 250,000/mm3 MCV 86 fL ภาวะดังกลาวเกิด จากสาเหตุใด 1. autoimmune 2. ขาด Vit B12 3. Hb ผิดปกติ 4. ขาด enzyme 5. Post hemorrhage 200. เด็กชาย 5 ป มีอาการเขาไดกับ down syndrome ผลการตรวจ karyotype ได 46,XY ความผิดปกติใดเขาได กับเด็กคนนี้ 1.deletion 2.mosaicsm 3.translocation 4.somatic mutation 5.undetected trisomy
  • 25. Basic Medical Science, March 2007 สหพันธสิตนักศึกษาแพทยแหงประเทศไทย National Council of Thai Medical Students (NCTMS) www.nctms.in.th -25- 200.เด็กชายชาวเขาอายุ 1ป ไมไดรับวัคซีน ไอมา2สัปดาห ไอมากเปนชุดๆ ไอจนตัวงอและหนาแดง ไอมาตอนกลางคืน ไมมีไข ฟงปอดปกติ ตรวจพบ Subconjunctival hemorrhage ถามวา อาการทาง Clinic เกิดการกลไกใด 1. การเปลี่ยนแปลง Normal flora 2. Mucosal inflammation 3. Increase mucous secretion 4. Pseudomembranous formation 5. Reduced mucociliary clearance 201. ชายอายุ 20 ป โดนงูเหากัด ไดรับเซรุมแกพิษงู มี อาการ urlicaria rash generalized lymphadenopathy , arthritis ขอใดลดลง 1. Serum complement 2. Absolute white blood cell 3. Serum IgG 4. Lymphocyte 5. ESR 203. จากการศึกษาการเกิด leptospirosis ในไทยป 2548 อัตราอุบัติการณ = 0.4 และ อัตราการตาย = 0.02 ตอ ประชาการแสนคน case fatality rate มีคาเทาไหร a. 0.0002 b. 0.004 c. 0.4 d. 0.02 ตอประชากรแสนคน e. 0.4 ตอประชากรแสนคน 204. ชาย 25 ป มีอาการปวดแสบใตลิ้นป ตอมาไปดื่ม alcoholอยางมากจึงมาโรงพยาบาลผลการทําGastroscope พบ Gastric mucosa บวม แดง Gastric biopsy พบ Neutrophil จงบอกกลไกที่ทําใหเกิดพยาธิสภาพ 1. Autoimmune 2. Direct mucosal toxicity 3. Stimulate acid secretion 4. Alcohol-induced Thiamine deficiency 5. Stimulate growth of H.pylori 205. เกิด Point mutation ที Ribosome Subunit 50 ของ Bacteria จะทําใหดื้อยาอะไรมากที่สุด 1. Cloxacillin 2. Ceftriaxone 3. Norfloxacin 4. Tetracycline 5. Erythromycin 206. ปวดชาตามปลายนิ้ว เมื่อโดนอากาศเย็น อากาศอุน แลวดีขึ้น ถามวา involve เสนเลือดใด 1. Large artery 2. Medium artery 3. Small artery 4. Capillary 5. Venule 207. ชายอายุ 50 ป เปนเบาหวานมา 15 ป มาร.พ. ดวย อาการขาบวมทั้งสองขางมา 2 เดือน ตรวจรางกาย BP 150/80 ตรวจปสสาวะ glucose 2+ protein 4+ RBC 2-3 cells/HPF WBC 1-2 cells/HPF granular cast 1-2/HPF ไตนาจะมีสภาพอยางไร 1. nodular glomerulosclerosis 2. minimal-change glomerulonephritis 3. membranous glomerulonephrtis 4. membrano-proliferative glomerulonephritis 5. focal sclering glomerulonephritis 208. หญิง 25 ป อาชีพทําสวน กินยากําจัดวัชพืช หลังจาก ทะเลาะกับสามี ตอมามีภาวะไตวายและการหายใจ ลมเหลว เกิดจากการไดรับสารพิษใด 1. paraquat 2. organophosphate 3. ethylene glycol 4. arsenical compound 5. sodium hypochlorite
  • 26. Basic Medical Science, March 2007 สหพันธสิตนักศึกษาแพทยแหงประเทศไทย National Council of Thai Medical Students (NCTMS) www.nctms.in.th -26- 209. นักศึกษาแพทยชั้นปที่ 4 ไดรับการตรวจ tuberculin test พบใหผลบวก ภาพรังสีทรวงอกปกติ การที่ภาพรังสี ทรวงอกปกติ เปนผลมาจากระบบภูมิคุมกันใด 1. antibiotic 2. helper T cell 3. B lymphocyte 4. Innate immunity 5. Complement activation 210. เด็กชายอายุ 5 ป เปน nephrotic syndrome ไดรับ prednisolone อยางตอเนื่องเปนเวลา 1 ป ควรหลีกเลี่ยง การให vaccine ใดในผูปวยรายนี้ 1. tetanus toxoid 2. varizella vaccine 3. influenza vaccine 4. hepatitis B vaccine 5. Japanese encephalitis vaccine 211.ชายอายุ 30 ป อาเจียนเปนเลือด ไดรับเลือด 30 cc มี อาการไข หนาวสั่น แนนหนาอก BP80/60 อาการดัง กลาวคืออะไร 1. anaphylaxis 2. citrate intoxication 3. bacterial endotoxic 4. immune complex reaction 5. antibody dependent cytotoxicity 212. ชายอายุ 60 ป มีอาการมือสั่นมา 2 เดือนเปนมากขึ้น ขณะพัก ลําตัว rigid แขนไมแกวง ยาที่ใชรักษาโรคนี้ตองมี ฤทธิ์อยางไรจึงจะดีที่สุดสําหรับโรคนี้ DA Cholinergic a. เพิ่ม เพิ่ม b. ลด ลด c. เพิ่ม ลด d. ลด เพิ่ม 213. ชายอายุ 25 ปจมน้ํา 5 นาที ไมรูสึกตัว ไมหายใจ หลังทํา PCR หัวใจเตน และหายใจได แตก็ยังใชเครื่องชวย หายใจอีก 20 ชั่วโมง ตอมารูสึกตัว ถามวา higher cortical function ใดเสีย 1. นับเลข 2. บวกเลข 3. อานหนังสือ 4. จําชื่อญาติ 5. จําชื่อโรงพยาบาล 214.หญิงอายุ 30 ป เปน SLE ไดรับการรักษาดวยยา corticosteroid มานาน เกิดภาวะแทรกซอน bacteria infection หลายครั้ง ภาวะแทรกซอนที่เกิดขึ้นเกิดจาก ความผิดปกติของกระบวนการใดในการกําจัดเชื้อกอโรค a. ingestion b. attachment c. chemotaxis d. degranulation e. lysosome production 215. ผูปวยหญิงมีอาการปสสาวะแสบขัด ตรวจปสสาวะ พบ E.coli ตองพบเชื้อจํานวนเทาใดในหนวย Colony forming unit / mLจึงจะถือวาเปนเชื้อกอโรค 216. หญิงอายุ 17 ป นั่งรถไฟไปเชียงใหมมีอาการฉี่เปน เลือด ผล LAB ........., Nitrite + หญิงคนนี้จะติดเชื้อใด 1. E.coli 2. Enterococcus Spp. 3. Trecomonas vaginalis 4. C.albicans 217. ผูปวยอายุ60ป มีปญหากินขาวแลวจุกหนาอก เปนมา3เดือน ตอนแรกกลืนขาวแลวจุก ตอนหลังกลืน ของเหลวไดอยางเดียว น้ําหนักลด 10กิโล สองกลองทาง ปากพบกอนที่หลอดอาหารสวนกลาง ถาตัดมาดูจะพบวา เปน a. adenoma b. adenocarcinoma c. squamous cell carcinoma d. hemangioma e. cystadenocarcinoma
  • 27. Basic Medical Science, March 2007 สหพันธสิตนักศึกษาแพทยแหงประเทศไทย National Council of Thai Medical Students (NCTMS) www.nctms.in.th -27- 218. Organophosphate มีกลไกเกิดพิษอยางไร 1) ลดการสราง Ach 2) ลดการหลั่ง Ach 3) ลดการทํางานของ Ach esterase 4) ลดการ reuptake Ach 5) ลด sensitivity Ach receptor 219. เด็กชายอายุ 1 ป เปนโรคทางพันธุกรรมชนิดที่มีภาวะ พรอง hydrolytic enzyme ถามวา enzyme ดังกลาวอยูใน organelle ใด A. acrosome B. lysosome C. ribosome D. microsome E. peroxysome 220. หญิงอายุ 45 ป ซีดและออนแรง ตรวจรางกายพบ Cardiac murmur ที่ pulmonaric valve และมีคา Hct 20% อยากทราบวา Cardiac murmur เกิดจากอะไร ก. หัวใจบีบตัวแรงขึ้น ข. ปริมาตรเลือดเพิ่มขึ้น ค. ความดันโลหิตสูงขึ้น ง. ความเร็วการไหลของเลือดเพิ่มขึ้น จ. ความตานทานหลอดเลือดเพิ่มขึ้น 221. หญิงอายุ35 ป มีกอนที่ทองดานขวาเมื่อผาตัดดูพบ กอนที่รังไขดานขวาเมื่อผากอนออกดูพบผมและฟนกอนนี้ คืออะไร ก) teratoma ข) cisrotoma ค) harmatoma ง) chondoma จ) cystadenoma 222. ผูหญิงอายุ50ป ทวมเล็กนอย เชื่องชา มีเสียงแหบ ... ตรวจพบ TSH ต่ํา ทําTRH stimulation test พบวาTSH เพิ่มขึ้นอยางชาๆ ถามวา อวัยวะใดผิดปกติ a. Thyroid gl. b. Thalamus c. Hypothalamus d. Anterior pituitary gl. e. Posterior pituitary gl. 223. เด็กชายอายุ 5 ปไดรับการวินิจฉัยเปน glioma ขนาด 4 cm ที่ right frontal lobe ไดรับการรักษาโดย224. การ ฉายรังสีจนครบพบวากอนยุบลงเล็กนอย cell มะเร็งมี กลไกใดทําใหดื้อตอการรักษา a. inactivation ของ p53 gene b. inactivation ของ bcl-2 gene c. inactivation ของ promoter of MDR-1 gene d. activation ของ BCL-xl gene e. activation ของ retinoblastoma gene 225. Blood pressure ลดลงจาก 110/70 เปน 100/70 เมื่อลุกขึ้นยืนเกิดจากสาเหตุใด 1. venous pooling ลดลง 2. stroke volumeลดงลง ใชยาใดรักษา 1. praziquentel 2. albendazol 3. metronodazol 4. ivermectin 5. amphotericin B